Anda di halaman 1dari 109

BAHASA InggRIS

Set 1
Reading Comprehension

Reading Comprehension is a part of the test. It is also influenced by language competence


(kemampuan bahasa), reading techniques (teknik membaca), dan reading skills
(kemampuan membaca). The most important thing from the three techniques is finding the
keywords from each questions. Therefore, here are some reading techniques that could help
you answering the questions.

A. SKIMMING
Skimming yaitu teknik membaca sekilas untuk mendapatkan inti dari bacaan. Teknik ini
digunakan untuk menyelesaikan soal yang menanyakan topik, main idea, atau judul dari
bacaan. Sebelumnya harus diketahui terlebih dahulu perbedaan judul, topik, dan
main idea dalam sebuah bacaan:
a. Judul (title) mewakili isi bacaan dan dianggap paling menarik bagi pembaca
untuk membaca sebuah teks. Judul yang ada belum tentu merupakan topik bacaan,
tetapi topik bacaan dapat menjadi judul bacaan.
b. Topik (topic) adalah inti masalah yang dibicarakan oleh penulis, baik dalam bentuk
kelompok kata maupun frasa (bukan kalimat). Topik tidak boleh terlalu khusus
(spesifik), maupun terlalu umum (general).
c. Pikiran pokok (main idea), sama seperti halnya topik, adalah inti masalah yang
dibicarakan penulis. Mencari main idea pun menjadi sesuatu yang cukup sulit
dirasa oleh sebagian besar pembaca. Perlu diingat bahwa dalam satu bacaan
hanya

1
terdapat satu pikiran pokok, yang letaknya bisa di awal atau di akhir paragraf
pertama. Main idea biasanya menekankan kepada order (urutan waktu dan
peristiwa), listing (daftar jumlah atau angka tertentu dalam bacaan), cause and
effect (pernyataan sebab-akibat melalui penggunaan kata penghubung atau
conjunctions), dan derajat perbandingan (comparison).

B. SCANNING
Scanning yaitu membaca sekilas namun cukup spesifik untuk mendapatkan informasi
tertentu saja sesuai yang ditanyakan dalam suatu bacaan. Umumnya, teknik ini digunakan
untuk menyelesaikan soal yang mengandung unsur 5W1H (what, where, when, who, why,
how) serta mengidentifikasi pertanyaan yang tidak sesuai dengan teks (true or false).

C. CLOZE READING
Cloze reading yakni membaca sekilas namun mendalam. Teknik ini digunakan umumnya
untuk menyelesaikan soal-soal yang berhubungan dengan kosakata (vocabulary) dan
bermacam kata jadian (derivatives).

D. VOCABULARY IMPROVEMENT
Dalam menjawab soal tentang vocabulary, penguasaan kosakata yang dimiliki
pembaca akan berbeda-beda, tergantung pada individu masing-masing dan
seberapa sering pembaca mengasah kosakata yang dimilikinya. Dalam bacaan, salah
satu cara untuk menjawab soal yang berhubungan dengan bacaan yakni dengan
melihat kalimat sebelum dan sesudahnya sesuai dengan konteks kalimat tersebut
disertai dengan penggunaan struktur tata bahasa yang tepat secara gramatikal.

CONTOH SOAL
The government published a report yesterday saying that we need to eat more healthily— more frui

Text 1: SPMB 2002

2
1. The topic of the text is ….
A. people’s reaction to suggestion on healthy eating habits
B. eating a healthy lunch in Central London
C. the government’s report on people unhealthy eating habits
D. fattening foods served at the Simpson’s restaurant
E. the advantages of eating fruits and vegetables
Pembahasan :
Teks ini diawali dengan kalimat yang menyatakan bahwa pemerintah telah
mengumumkan tentang program makan makanan sehat bagi masyarakat. Adapun
sebagian besar kalimat dalam paragraf ini mengetengahkan tentang komentar dari
masyarakat yang dengan kata lain merupakan reaksi dari masyarakat terhadap kebijakan
baru pemerintah tersebut. Maka, dapat disimpulkan bahwa jawaban yang paling tepat
untuk pertanyaan tersebut adalah: people’s reaction to suggestion on healthy eating habits.
Jawaban: A

2. What does the restaurant manager think of the report?


A. People don’t understand the advice given.
B. The government’s advice is inconsistent.
C. People are more concerned about losing weight.
D. Fattening foods are more delicious.
E. People will welcome the government advice.
Pembahasan :
Mengacu pada pertanyaan yang diajukan, maka fokus kita akan langsung menuju
kepada restaurant manager sebagai kata kunci dari pertanyaan tersebut. Jika
ditelaah dengan baik, maka dapat disimpulkan bahwa restaurant manager tersebut
tidak mendukung kebijakan pemerintah yang menurutnya berubah-ubah dan
cenderung tidak konsisten. Maka, dapat disimpulkan bahwa jawaban yang paling tepat
untuk petanyaan tersebut adalah: The governments advice is inconsistent.
Jawaban: B

Text 2: SPMB 2004


Throughout history, people have been the victims of the pickpockets. Today, pickpocketing is one of t
During the eighteenth century, pickpockets were hanged in England. Large crowds of people would gat

3
people attentivelly watching the hanging of a pickpocket, other pickpockets skillfully stole the m
Police officials say that the most efficient pickpockets come from South America. Many of these
Even the most well-dressed, respectable person can be a pickpocket. Some of the favorite places
To avoid being the victim of the pickpocket, it is important to be very cautious and alert when in

3. The best title for the passage would be ….


A. Pickpockets are Well-dressed
B. Jingle Bells Schools
C. Pickpockets
D. The Hanging of Picpockets
E. The History of the Pickpockets
Pembahasan :
Dalam menentukan sebuah title (judul), pilihlah jawaban yang paling general dan mampu
menggambarkan bacaan tersebut secara umum. Maka, title (judul) yang paling mewakili
bacaan tersebut adalah (C) pickpockets karena jawaban yang lain bersifat spesifik.
Jawaban: C

4. The hanging of pickpockets were not continued because ….


A. they were large crowds of people
B. It was against humanity
C. the police were not hanging a pickpocket
D. it was not effective to stop pick-pocketing
E. he was caught by the police
Pembahasan :
Kata kunci dari pertanyaan ini adalah hanging. Mengacu pada bacaan yang ada, kita
ketahui bahwa menggantung seorang pencopet bukanlah sebuah hukuman yang
bijaksana karena saat orang-orang berkerumun menyaksikan sang pencopet dihukum,
maka pencopet lainnya akan leluasa mencopet di tengah kerumunan. Hal tersebut
membuat pelaksanaan hukuman mati di rasa menjadi tidak efektif.
Jawaban: D

4
5. Pickpocketing is a … crime.
A. an old
B. a modern
C. a temporary
D. a contemporary
E. a temporary old
Pembahasan :
Pada kalimat awal di paragraf pertama, pembaca seolah diceritakan kembali tentang
sejarah lahirnya pencopet sebagai salah satu kejahatan tertua yang pernah ada.
Bahkan hampir di seluruh dunia, jutaan orang telah menjadi korbannya.
Jawaban: A

6. “… other pickpockets skillfully stole the money from the


spectators!” The similar meaning of the italic word is ….
A. people
B. victims
C. South American
D. audiences
E. passangers
Pembahasan :
Spectators dalam kutipan kalimat tersebut bermakna penonton (audiences) yang sedang
menyaksikan pencopet yang sedang menjalani hukuman gantung.
Jawaban: D

7. South American pickpockets are the most efficient ones because ….


A. they are well-trained
B. they are handsome
C. they are well-dressed
D. they are respectable
E. they are persistent
Pembahasan :
Kata kunci dari pertanyaan tersebut adalah South America. Kata kunci tersebut dapat
ditemukan pada paragraf ketiga yang di dalamnya membahas kehidupan para
pencopet yang tinggal di Amerika Selatan (Amerika Latin). Hal yang membuat para
pencopet tersebut menarik adalah ketika terdapat sekolah khusus bagi pencopet
disana sehingga mereka menjadi orang yang terlatih (well-trained) dalam menjalankan
aksinya.
Jawaban: A

5
BAHASA InggRIS
Set 2 Report Text

Purpose of the Text :


To inform the readers about natural or social phenomena; to describe the way things
are.
Tujuan Penulisan :
Menyampaikan informasi tentang suatu hasil pengamatan secara apa adanya.

Generic Structure (Struktur Umum) :


1. General Classification: pernyataan umun yang menerangkan subjek laporan, keterangan,
dan klasifikasinya.
2. Descriptions: pembuatan informasi tentang ciri-ciri umum/generalisasi yang dimiliki
subjek seperti perilaku, fisik, psikologis, kualitas, ciri-ciri khas.

Language Features (Ciri-ciri Bahasa):


1. Menggunakan Simple Present Tense karena menyatakan fakta/kebenaran umum
2. Hasil pengamatan, observasi, penelitian atau studi tentang benda, binatang, orang
maupun tempat

1
CONTOH SOAL

Text 1: UMPTN 1998/R-A/14


Whales are the largest animals on earth. Bigger than elephants, they may grow 95 feet long, and we
Although whales live in the oceans and swims like a fish, they are not fish. They are mammals, like
Whales live in all the oceans. In the winter, some of them go to the warm water to breed and in the
Whales have few enemies. Only human beings and the killer whales that attack whales.
And whales do not seem to fight among themselves. They usually live from 20−30 years.

1. The difference between whales and fish is that whales ….


A. do not have to come to the surface to get fresh air
B. deliver their young like elephants do
C. will die if they can not get oxygen in the water
D. are mammals that are cold-blooded
E. can easily breathe when they are under water
Pembahasan :
Pada paragraf kedua dalam bacaan, dikatakan bahwa paus adalah mamalia terbesar
yang hidup di laut. Namun, sekalipun mereka tinggal dan hidup di air, mereka tetap
membesarkan anak yang mereka lahirkan dan bukan telur karena mamalia adalah
hewan yang melahirkan, seperti halnya sapi maupun gajah dalam mengasuh anak mereka.
Jawaban: B

2. A suitable title for the text is ….


A. Large Animals on Earth
B. Difference Betweeen Whales and Elephants

2
C. Whales, the Biggest Animals on Earth
D. Similarity Between Fish and Whales
E. The Life of Whales in the Oceans
Pembahasan :
Dalam bacaan tersebut, dari awal hingga akhir paragraf menggambarkan serta
menyampaikan informasi tentang mamalia terbesar yang hidup di laut, yakni paus.
Maka dari itu, jawaban yang paling tepat untuk soal ini adalah (C) Whales, the Biggest
Animals on Earth.
Jawaban: C

3. Which of the following is TRUE about the whales?


A. Whales are generally 95 feet long and weigh 150 tons.
B. Big animals living in the ocean tend to be their enemies.
C. Like fish, whales can live in all kinds of water.
D. Their life-span is usually longer than human beings.
E. A new-born baby whale may weight one-fiftieth of its mother.
Pembahasan :
Berdasarkan bacaan di atas, maka jawaban yang paling tepat adalah (A) yang terdapat
pada kalimat kedua di paragraf pertama. Sementara pilihan lainnya dalam soal tersebut
tidak sesuai dengan yang terdapat dalam bacaan.
Jawaban: A

4. Whales occasionally live in the warm water ….


A. to get more foods
B. for reproduction
C. to avoid winter
D. to get fresh air
E. to feed their babies
Pembahasan :
Pada paragraf ketiga baris kedua dikatakan “… in the winter, some of them go to the
warm water to breed” yang sesuai dengan pilihan jawaban (B) karena breed =
reproduction (berkembang biak).
Jawaban: B

5. From the text we may conclude that ….


A. a whale can both eat squid and plankton
B. toothed whales chew their foods before swallowing it
C. the two types of whales live in the different waters
D. baleen whales swallow the water containing the plankton
E. each type of whale eats a specific kind of foods

3
Pembahasan :
Berdasarkan informasi yang terdapat pada paragraf ketiga, maka dapat disimpulkan
bahwa baik paus yang memiliki gigi (toothed whales) dan yang tidak memiliki gigi (baleen
whales) memiliki makanan yang tersendiri sesuai dengan bentuk tubuh dan keunikan
yang mereka miliki.
Jawaban: E

Text 2: SPMB 2006/R-II/610

The killer sea waves known as Tsunamis are so quiet in their approach from a far, so
harmless, that until recently their history has been one of surprise attack.
Out in the middle of the ocean, the distance between tsunami wave crests can be
100 miles and the height of the wave have no more than three feet: sailors can ride one
and suspect nothing. At the shoreline, the first sign is often an ebbing—a retreat but
rather a gathering of forces. When the great waves finally do strike, they rear up and hit
harbor and coast, causing death and damage.
These seismic sea waves—or tidal waves—as they are sometimes called, bear no
relation to the moon or tides. And the word “tsunami”, Japanese for “harbor wave”, related
to their destination rather than their origin. The causes are various: undersea or coastal
earthquakes, deep ocean avalanches or volcanism. Whatever the cause, the wave motion
starts with a sudden move like a hit from a giant paddle that displaces the water. And
the greater the undersea hit, the greater the tsunami’s damaging power.
In 1883, Krakatoa volcano in the East Indies erupted, and the entire island collapsed
in 820 feet of water. A tsunami is a tremendous forces bouncing around Java and
Sumatera, killing 36.000 people with walls of water that reached 115 feet in height.
In 1946, a tsunami struck first near Alaska and then, without warning, hit Hawaiian
Islands, killing 159 people and causing millions of dollars of damage. This led to the creation
of the Tsunami Warning System, whose nerve centre in Honolulu keeps around—the—
clock vigil with the aid of new technology. If the seismic sea waves are confirmed by the
Honolulu centre, warnings are transmitted within a few hours to all threatened Pasific points.
Tsunamis have been deprived of their most deadly sting—surprise.

6. The writer describes in details ….


A. how harmless the tsunamis seems to be for sailors
B. what damage the tsunami caused in Japan
C. when the tsunami tends to attack unexpectedly
D. why tsunamis are called the killer sea waves
E. where tsunamis have been most severe in their attack

4
Pembahasan :
Dalam bacaan tersebut, penulis ingin menyampaikan betapa dahsyatnya bencana alam
tsunami yang tidak disangka-sangka kedatangannya. Pada awal paragraf dikatakan,
munculnya tsunami terjadi secara tiba-tiba tanpa adanya tanda-tanda alam yang disadari
oleh manusia. Maka, jawaban yang paling tepat untuk pertanyaan tersebut adalah C.
Jawaban: C

7. Why are tsunamis seemingly harmless when they start?


A. The waves out in the ocean were not alarmingly high.
B. The ebbing of the water leave fish stranded and slapping on the bottom.
C. The great waves strike, reaching up and hitting harbor and coast.
D. The waves approaching from a far gather forces and move the water away from
the land.
E. The seismic sea waves bear no relation to the moon or tides.
Pembahasan :
Pada paragraf kedua dikatakan bahwa jauh di lautan sana, gelombang tsunami
berjarak sekitar 100 mil dan tingginya tidak lebih dari tiga meter. Hal tersebut tidak
menimbulkan kekhawatiran bagi para pelaut yang menganggap hal tersebut adalah hal
yang wajar. Di tepian pantai pun gelombang yang terjadi tidaklah dahsyat sehingga
tsunami tidak terlihat menakutkan ketika mulai terjadi. Dilihat dari pilihan jawaban
yang ada, maka jawaban (A)-lah yang paling tepat karena awalnya terlihat sebagai
gelombang biasa maka kedatangannya sangatlah mengejutkan.
Jawaban: A

8. The following are possible causes for seismic waves, EXCEPT ….


A. earthquakes under the oceans
B. underwater avalanches
C. the rise and fall of the sea levels
D. ocean volcanoes
E. seashore earthquakes
Pembahasan :
Yang bukan merupakan penyebab dari terjadinya tsunami adalah naik dan turunnya
permukaan air laut karena air laut yang pasang dan surut adalah hal yang umum terjadi di
laut dan tidak berpotensi menyebabkan tsunami; seperti pada jawaban (C).
Jawaban: C

9. Which of the recorded tsunami in the text was the worst? The one that hit ….
A. The East Hindies
B. The Island of Java
C. Alaska

5
D. The Hawaiian Islands
E. Honolulu
Pembahasan :
Berdasarkan keterangan pada paragraf keempat dalam bacaan, disebutkan bahwa salah
satu bencana tsunami yang terdahsyat pernah terjadi di sekitar kepulauan Jawa
dimana bencana tersebut telah menelan korban jiwa lebih dari 36.000 orang dan
gelombang yang sangat tinggi mencapai ketinggian 115 kaki.
Jawaban: B

10. The danger of tsunamis has been reduced by ….


A. establishing local nerve centre
B. the countries around the Pasific Rim
C. close control from a new warning centre
D. warning the nerve centre of seismic waves
E. the Tsunami Warning System at the bottom of the sea
Pembahasan :
Bahaya tsunami telah menyadarkan banyak pihak akan pentingnya mengetahui gejala
tsunami dari awal kemunculannya. Berdasarkan bacaan di paragraf kelima, banyak negara
kemudian saling bekerjasama untuk mendirikan pusat peringatan bencana tsunami
dan mengawasi dari dekat secara langsung gejala yang umumnya muncul menjelang
terjadinya tsunami untuk menghindari korban jiwa yang lebih banyak lagi.
Jawaban: C

6
BAHASA InggRIS
Set 3 Discussion Text

Purpose of the Text :


To present information and opinions about more than one side of an issue
(“For/Pros”) and (“Against/Cons”)

Tujuan Penulisan :
Mengajak pembaca mengambil sikap tertentu terhadap sebuah issue atau masalah.
Pembaca diharapkan dapat mengidentifikasi isu tersebut dan mengulas pendapat yang
mendukung (Pro) maupun yang menentang (Cons) sebelum memberikan rekomendasi
terhadap isu tersebut.

Generic Structure (Struktur Umum) :


1. Issue: topik perhatian/isu utama
2. Arguments: alasan/pendapat yang mendukung gagasan pokok yang diikuti points dan
elaboration (uraian)

Language Features (Ciri-ciri Bahasa) :


1. Menggunakan Simple Present Tense
2. Conjunctions (Kata Penghubung)
3. Related verbs and Modal Constrastive

1
CONTOH SOAL

Text 1: SPMB 2004/R-I/440

The view that women are better parents than men has shown itself to be true
throughout history. This is not to say that men are not of importance in child-rearing;
indeed, they are most necessary if children are to appreciate fully the roles of both
sexes. But women have proven themselves superior parents as a result of their
conditioning, their less aggressive nature and generally better communication skills.
From the time they are little, females learn about nurturing. First with dolls and
later perhaps with younger brothers and sisters, girls are given the role of career. Girls
see their mothers in the same roles so it is natural that they identify this as female
activity. Boys, in contrast, learn competitive roles far removed from what it means to
nurture. While boys may dream of adventures, girls conditioning means they tend to see the
future in terms of raising families.
Girls also appear to be less aggressive than boys. In adulthood, it is men not women,
who prove to be the aggressors in crime and in war. Obviously, in raising children, a
more patient, gentle manner is preferable to a more aggressive one. Although, there
certainly exist gentle men and aggressive women, by and large, female are less likely to
resort to violence in attempting to solve problems.
Finally, women tend to be better communicator than men. This is shown in intelligence
test, where females, on average, do better in verbal communication than males. Of course,
communication is of utmost important in rearing children, as children tend to learn from and
adopt the communication styles of their parents.
Thus, it is all very well to suggest a greater role for men in raising children, let us
not forget that women are generally better suited to the parenting roles.

1. The main information of the text is about ….


A. the importance of men and women in child rearing
B. the reluctance of men to play the role of parents
C. the reasons why women are better parents than men
D. the superior of women in family
E. the different attitudes of men and women as parents
Pembahasan :
Kalimat ditandai oleh kata penghubung yang dapat menunjukkan daftar (listing)
seperti: first, later, finally serta kata penghubung lainnya yang menunjukkan beberapa
perbandingan. Dari bacaan tersebut jelas terlihat adanya perbandingan serta alasan-
alasan yang timbul di masyarakat bahwa wanita memang lebih baik sebagai orangtua
dibanding pria dengan segala kelebihan dan kekurangannya.
Jawaban: C

2
2. As parents, women in general play a more important role than men because they are ….
A. not aggressive at all
B. good communicators
C. superior human beings
D. experienced in raising children
E. capable of solving problems
Pembahasan :
Pada kalimat terakhir pada paragraf pertama dikatakan bahwa wanita mempunyai peranan
yang lebih luas dan lebih banyak karena wanita umumnya mempunyai kemampuan
dalam berkomunikasi dan mengkomunikasikan segala sesuatunya dengan lebih baik
dibanding pria.
Jawaban: B

3. Most women are good mothers because they ….


A. have brothers and sisters with whom they play
B. had to learn about nurturing when they were children
C. have never dreams of adventure like boys
D. are not to learn about competitive roles
E. have known the role of career since childhood
Pembahasan :
Sejak kecil wanita telah diajarkan baik secara langsung maupun tidak langsung oleh
lingkungan, terutama dari ibu mereka, mengenai apa yang harus mereka lakukan di masa
depan, terutama tentang bagaimana cara merawat dan mengasuh seorang anak.
Tanpa disadari, setiap wanita telah mempunyai naluri keibuan sejak mereka masih kecil,
entah karena terbiasa bermain dengan saudara maupun dengan meniru perilaku ibu
mereka dalam mengasuh anak; terbukti dengan lebih mampunya wanita dalam
merawat anak dan keluarga dibanding pria pada umumnya.
Jawaban: E

4. The following are the general characteristics of men, EXCEPT ….


A. aggressive
B. adventures
C. impatient
D. irresponsible
E. competitive
Pembahasan :
Aggressive ditunjukkan dalam kalimat “Girls are also appear to be less aggressive
than boys”; adventurous ditunjukkan dalam kalimat “Boys may dream of adventures”;
impatient ditunjukkan dalam kalimat “It ismen, notwomen, tobetheaggressor.”; dan
competitive dalam kalimat “Boys, in contrast, learn competitive roles.” Dari pilihan
kosakata yang ada, maka

3
yang tidak termasuk karakteristik seorang pria secara umum adalah (D) Irresponsible atau
tidak bertanggung jawab. Karena tidak semua pria adalah pria yang tidak bertanggung
jawab pada berita tersebut
Jawaban: D

5. According to the text, which of the following statement is TRUE about parents in
general?
A. Children love their mothers more than they love their fathers.
B. All fathers tend to be aggressive and violent towards their children.
C. Mothers play a greater role in the education of their children.
D. Even gentle fathers are unable to communicate with children.
E. There is no communication between fathers and their children.
Pembahasan :
Secara keseluruhan, teks tersebut menggambarkan tentang betapa hebatnya peranan
seorang wanita dalam kehidupan keluarga mereka. Meskipun pria juga mempunyai
andil yang cukup besar, namun peran seorang ibu tidak dapat tergantikan oleh
siapapun. Karena itu, jawaban yang paling tepat untuk pertanyaan tersebut adalah (C).
Jawaban: C

Text 2: SBMPTN 2014


Did you ride your bike to school when you were a kid? A generation ago most kids rode, walked or c
Parents choose to drop their kids at school for a number of reasons—mostly do with safely and conv
Researchers suggest at least a third of Australian children aged 9−16 years are not getting the amou

4
6. The author develops some ideas in paragraf 2 by ….
A. describing parents’ chauffeuring followed by its effects
B. explaining reasons for chauffeuring and their advantages
C. discussing the function of chauffeuring and the impacts
D. arguing for chauffeuring practice for children’ safety
E. exposing how parents chauffer and its drawbacks
Pembahasan :
Menurut paragraf kedua dari bacaan di atas, para orangtua memilih untuk menyetir
dan mengantarkan langsung anak mereka ke sekolah karena berbagai alasan. Di
antaranya karena alasan keamanan dan kenyamanan. Namun, tanpa mereka sadari,
dengan selalu mengantarkan anak mereka ke sekolah membuat anak-anak kehilangan
waktu untuk bersosialisasi dan juga berolahraga di lingkungan sekitar; dan itulah
yang menjadi kelemahannya.
Jawaban: E

7. By writing the sentence ”… chauffeuring your kids to school every day could mean
they are missing out on much-needed exercise and other life skills.” (paragraph 2 lines 2-
3); the author implies that ….
A. taking kids to school makes them deprived individuals when they grow up
B. kids given lift to school likely lose vital social and physical advantages
C. schooling means not only learning in classes but also socializing with others
D. parents spoil their kids’ future social and physical life by giving them a lift
E. when a child needs physical and social training, parents should facilitate them
Pembahasan :
Berdasarkan petikan kalimat tersebut, dapat disimpulkan bahwa selain memiliki sisi
positif dari pihak orangtua, namun ternyata terdapat sisi yang negatif bagi para anak.
Diantar-jemput setiap hari ke sekolah membuat mereka kehilangan kesempatan untuk
bersosialisasi dengan lingkungan dan menggerakkan tubuh.
Jawaban: B

8. Dr. Garrard statement “… where children are just active as a part of everyday life, they
do not have to be sporty.” (paragraph 3 line 5) may be best restated that ….
A. children who are active do not automatically mean they will be good sport
B. being muscular should not be the aim of children who are naturally active
C. children’ physical fitness is not closely related with their daily activities
D. activeness in children does not mean to make these children physically fit
E. when naturally active, children need no more scheduled sports activities
Pembahasan :
Pada kalimat di atas, dengan kata lain dapat dikatakan bahwa ketika anak-anak sudah

5
terbiasa untuk bergerak secara aktif, maka sesungguhnya mereka tidak perlu lagi
melakukan olahraga secara rutin. Karena dengan selalu bergerak setiap hari sama dengan
melakukan olahraga secara rutin. Berdasarkan pilihan jawaban diatas, maka jawaban yang
paling tepat untuk pertanyaan tersebut adalah pada pilihan (E).
Jawaban: E

9. The part following the passage above most likely contains information on ….
A. advice to parents for their children to have enough physical activities
B. the decreasing trend of children to do physical activities at their will
C. parental motives behind chauffeuring their children to school
D. reason for children not to do fun and incidental activities
E. effects of having children not to be given a lift to school
Pembahasan :
Pada paragraf kedua dalam bacaan tersebut dikatakan bahwa orangtua memiliki banyak
motif atau alasan untuk mengantarkan sendiri anak mereka ke sekolah. Maka pilihan
jawaban yang paling tepat adalah pilhan jawaban (C).
Jawaban: C

6
BAHASA InggRIS
Set 4
Analytical and Hortatory Exposition

Purpose of the Text :


To persuade the readers that something should or shouldn’t be the case.

Tujuan Penulisan :
Memberikan pendapat/ide dari penulis tentang suatu topik atau permasalahan

Generic Structure (Struktur Umum) :


1. Menggunakan Simple Present Tense
2. Menggunakan connectives/transitions
3. Menggunakan action verbs and adjectives

Terdapat dua jenis eksposisi :


1. Analytical Exposition (Eksposisi Analisis)
a. Penulis memberikan opini dan tidak berusaha membujuk pembaca untuk
menjadi pro/kontra.
b. Nama lainnya adalah Argumentative.
c. Umumnya berisi tentang permasalahan umum seperti flood, traffic jam,
pollution, etc.
d. Generic Structure (Struktur Umum):

1
1. Thesis: pernyataan penulis tentang suatu kasus
2. Argument: inti/poin perbincangan dan penjelasan dari poin/inti
3. Reiteration: penguatan dari pernyataan

2. Hortatory Exposition
a. Penulis memberikan opini dan berusaha membujuk pembaca untuk pro/kontra.
b. Nama lainnya adalah Persuasive.
c. Generic Structure (Struktur Umum):
1. Thesis: pernyataan penulis tentang suatu kasus
2. Argument: alasan tentang adanya kepedulian dan mengarah ke saran/
rekomendasi
3. Recommendation: pernyataan alasan/bagaimana sesuatu seharusnya
dilakukan/tidak dilakukan

CONTOH SOAL

Text 1: SBMPTN 2013 Kode Soal 613

Passage A
School is necessary because it, first, makes communication with diverse people essential,
as parents do not choose where their children go, and secondly, homeschooling and
extracurricular activities connected to it cannot bring that diversity, for the attending group
is self-selecting, rather than ‘unfiltered mixture’. I believe that none of the two assumptions
is warranted nor true.
Inthefirstplace, parents still selectschoolsfortheirchildrenonthebasisofcommonvalues,
cultures and achievements. As such, public schools then offer obvious misrepresentations of
the society.
Not only that, I think it’s non-sense that homeschooling should somehow get rid
of diversity from socialization in sports teams or other clubs. What members of a basketball
team or a debate club share is not race, religion, nor income bracket: it is the desire to
participate.

Passage B
Children can perhaps be best described as beings of potential during their schools years.
Each child is in the process of discovering their talents and having them be cared for in
an attempt to best reach self-actualization.
Under the homeschooling system parents become much more exclusive mentors
to a child, and this can be problematic. Although parents are generally aware of what
their children is capable of, their evaluations are not always precise.

2
This could be for a number of reasons. First, it may due to the fact that children are predominantly

1. The common theme underlying both passages above is ….


A. arguments against home school education practices
B. evidence of several problems with homeschooling
C. reasons why homeschooling should be eliminated
D. failure which is associated with homeschooling
E. formal school and home school education
Pembahasan :
Pada passage A terdapat kalimat yang mengatakan bahwa “… I think it’s non-sense
that homeschooling should somehow get rid of diversity from socialization in sports teams
or other clubs.” Begitu pula sebaliknya pada passage B ditemukan tentang berbagai kontra
terkait dengan adanya homeschooling dalam bidang pendidikan. Sehingga kalimat yang
paling tepat mencakup kedua passage tersebut adalah jawaban (A).
Jawaban: A

2. Which of the following statements reflects a fact mentioned in either passage above?
A. Joining a social club is a matter of wish to participate.
B. Interacting with other individuals is very important.
C. Children are individuals with inborn capacity.
D. Sports teams are a form of social unit.
E. Parents play a role as a private guru.
Pembahasan :
Melalui kalimat “Whatmembersofabasketballteamoradebateclubshareisnotrace, religion,
nor income bracket: it is the desire to participate.” Pada passage A, dapat dikatakan bahwa
faktanya seringkali bukan hanya sekadar mengikuti sebuah klub atau menjadi bagian
dari sebuah tim yang diinginkan seorang anak, namun lebih kepada keinginannya
untuk berpartisipasi dan bersosialisasi dengan lingkungan mereka sendiri. Dan hal
tersebutlah yang tercermin pada kedua bacaan di atas.
Jawaban: A

3. The idea in passage B which is different from that in passage A is ….


A. thoughts that schools are a medium for individual to socialize freely
B. the need to join social events not diversity that homeschoolers seek
C. parents limited understanding of their children’s potentials and talents

3
D. parents personal preference to a certain type of school for their children
E. wrong assumptions on both home school education and homeschooling
Pembahasan :
Pada passage B dikatakan bahwa orangtua cenderung memiliki pemahaman yang
terbatas terhadap potensi serta bakat yang dimiliki oleh anak-anak mereka. Sehingga,
seringkali apa yang menjadi keinginan orangtua menjadi tidak sejalan dengan bakat
yang dimiliki oleh anak-anak mereka. Maka, jawaban yang paling mencerminkan
passage B adalah jawaban (C).
Jawaban: C

4. The best summary of both passages is ….


A. ideas of school and home school learning are wrong; yet, school is better
B. schools are the right medium for socializing; yet, parents are still selective
C. there is no rational support to believe education at home and at school
D. children are going their ways to develop their talents and personality
E. parents and teachers make inaccurate evaluation on children’ s talents
Pembahasan :
Kesimpulan yang paling tepat untuk kedua bacaan di atas, terlepas dari permasalahan
homeschooling dan bakat yang cenderung salah diarahkan, adalah sekolah merupakan
media yang paling tepat bagi anak uuntuk bersosialisasi; meskipun tidak dapat
dipungkiri bahwa orangtua pun masih tetap selektif dalam memilih sekolah yang
terbaik bagi mereka.
Jawaban: B

5. If a child’s parents are not able to identify the child’s talents comprehensively, a
home school child’s basketball talent will ….
A. never be revealed until the right expert can reveal the child’s talent
B. grow normally because somebody’s talent is not affected by anyone else
C. grow little and later it may stop growing at all as there is a little stimulation
D. remain undeveloped because none is able to reveal the child’s potential
E. still grow in so far he/she has the need to partake in a basketball team
Pembahasan :
Jika orangtua tidak mampu mengenali bakat dan talenta yang dimiliki dengan baik,
maka bakat tersebut akan menjadi hal yang sia-sia dan tidak dapat berkembang
karena tidak ada yang mampu mengembangkan bakat yang dimiliki selain individu itu
sendiri.
Jawaban: D

4
Text 2: SBMPTN 2013-TKDU Kode Soal 325

Passage A
For those of you now eyeing your cell-phones suspiciously, it is worth nothing
that both the National Cancer Institute (NCI) and the World Health Organization (WHO) say
there is no evidence to support the assertion that cell-phones are a public health threat.
But a number of scientists are worried that has been a dangerous rush to declare cell-
phones safe, using studies they feel are inadequate and too often weighted towards the
wireless industry interests. An analysis published by University of Washington neurologist
Henry Lai determined that far more independent studies than industry—funded studies
have found at least some type of biological effects from cell-phone exposure.
A strong link between mobiles and cancer could have major public-health implications.
As cell-phones make and take calls, they emit low-level-radio-frequency (RF radiation).
Stronger than FM radio signals, these RF waves are still a billionth the intensity of known
carcinogenic radiation like X-ray. (www.time.com)

Passage B
A study published in the journal Bioelectromagnetics reported that no statistically
significant change in the incidence of brain cancers in men and women in England
between 1998 and 2007, a time when cell-phones use increased dramatically.
But now, new work published this week in the Journal of the American Medical
Association says there is an identifiable effect of cell-phones use in the brain, but it is
too early to tell what, if anything, that effect means health-wise.
There has been a lot of controversy of whether cell-phones could increase the
temperature of the brain, which in turn could affect energy requirements.
Based on our study, we really cannot infer whether this is a bad or could even
have potentially good applications so that—our finding doesn’t illuminate or enlighten
that very important question or whether cell-phone exposure could have detrimental effects.
(www. npr.org)

6. Which of the following reflects opinions mentioned in the both passages?


A. There was no increase of brain cancer patients in men and women between 1998-
2007.
B. Statements by NCI and WHO on safe use of cell phones have led to a
dangerous rush.
C. There is some type of biological effects from mobile phone exposure.
D. Radiation emitted by X-rays is carcinogenic.
E. Mobile phones emit deadly radiation.

5
Pembahasan :
Pada kedua bacaan tersebut sama-sama memiliki keterkaitan yakni keduanya menyatakan
bahwa terdapat beberapa efek biologis akibat terkena paparan radiasi dari
penggunaan telepon selular yang berlangsung terus-menerus.
Jawaban: C

7. Information in both passages may lead to a hypothesis that ….


A. uncontrolled use of mobile phones will increase carcinogenic stuff in user’s brain
B. regular use of mobile cell-phones will increase biological defects not just in the brain
C. the increasing use of mobile phones will result in detrimental effects of their users.
D. careless use of mobile phones will increase levels of brains energy consumptions.
E. frequent use of mobile phones will increase the radiation accumulation in the brain.
Pembahasan :
Hypothesis adalah suatu prediksi atau perkiraan atas sesuatu yang sedang diamati
saat ini. Dalam hal ini, penggunaan telepon selular secara terus-menerus tanpa disadari,
cepat ataupun lambat, akan memberikan dampak yang sangat besar bagi kesehatan
terutama pada otak. Radiasi yang timbul akibat penggunaan telepon selular secara
berkala akan mengakibatkan kerusakan pada otak, syaraf, atau bahkan kanker otak.
Jawaban: E

8. The topic discussed in both passages above is ….


A. the increase in the brain cancer incidences by the mobile phones radiations
B. inconclusive empirical findings on the impact of the use of mobile phones
C. recent research findings on radiation by mobile phones on brain damage
D. arguments against frequent uses of mobile phones for health reasons
E. issues on the impact of using mobile phones on one state’s health
Pembahasan :
Topik yang terdapat pada kedua bacaan di atas adalah tentang hasil penelitian yang
dilakukan baru-baru ini yang mana hasilnya cukup mencengangkan yakni betapa besar
bahaya penggunaan telepon selular bagi otak, bahkan dapat menyebabkan kanker dan
penyakit lainnya.
Jawaban: C

9. Both passages are similar in content in terms of addressing ….


A. relation between mobile phones and health
B. recent research evidence on cell-phones uses
C. mobile phones potential effects of mobile phone use on users’ brain
D. mobile phone use in triggering the brain cancer
E. finding the research on the use of mobile phones

6
Pembahasan :
Kedua paragraf di atas memiliki persamaan yakni membahas efek yang ditimbulkan oleh
telepon selular terhadap otak penggunanya.
Jawaban: C

7
BAHASA InggRIS
Set 5 Present Tense

Present Tense adalah tenses yang digunakan untuk menunjukkan kejadian yang
berlangsung pada saat ini, baik yang sedang terjadi maupun yang telah selesai dan
masih dilakukan hingga saat ini.

Secara umum, Present Tense terbagi menjadi empat bagian, yakni:


A. SIMPLE PRESENT
a. Rumus dasarnya adalah S + Verb 1 atau S + Verb 1+ -s/-es
b. Digunakan untuk:
1. Menyatakan kebiasaan yang sering dilakukan (Habitual Actions)
Contoh: Ali always goes to school every day.
2. Menyatakan Kebenaran Umum (General Truth)
Contoh: The sun rises in the East and sets in the West.
3. Menyatakan Jadwal
Contoh: Our English class starts at 8 o’clock every Friday.
c. Adverb of frequency yang sering digunakan adalah: always, usually, often,
sometimes, rarely, seldom, ever, never, etc.

B. PRESENT CONTINOUS TENSE


a. Rumus dasarnya adalah S + to be + Verb-ing
b. Digunakan untuk menyatakan peristiwa yang terjadi tepat pada saat ini (sekarang)

1
c. Adverbs of time yang sering digunakan adalah: now, at this time, at this moment, etc.
d. Contoh: I am chatting with my foreign friend right now.

C. PRESENT PERFECT TENSE


a. Rumus dasarnya adalah S + Have/ Has + Verb 3
b. Digunakan untuk menyatakan suatu peristiwa atau kejadian yang telah selesai
terjadi/dilakukan pada waktu lampau (yang mana waktunya tidak ditentukan) dan
masih ada hubungannya dengan waktu kini (sekarang)
c. Adverb of time yang sering digunakan: already, yet, for, since, lately/recently, never,
once, twice, three times, etc.
d. Contoh: I have already done my homework.
e. Bisa digunakan untuk for dan since.
Contoh: Jane has lived there since 2001.

D. PRESENT PERFECT CONTINOUS TENSE


a. Rumus dasarnya adalah S + Have/ Has + been + Verb-ing
b. Digunakan untuk menyatakan kejadian yang telah dilakukan di waktu lampau
dan masih dilakukan sampai sekarang.
c. Umumnya terdapat keterangan durasi waktu yakni dengan menggunakan
kata hubung for dan since.
d. Contoh: People have been waiting for him for three hours.

CONTOH SOAL

1. “When are you going to tell your sister the good news?”
“When she … from her business trip. “ (UMPTN/2000/R-C/12)
A. will be returning
B. is going to return
C. returns
D. is returning
E. had returned
Jawaban: C
Pembahasan :
Dia akan memberitahu kakaknya tentang berita baik tersebut, segera setelah dia kembali
dari perjalanan bisnisnya. Karena dari awal kalimat pertanyaan berbentuk Present Tense
dan menunjukkan jadwal kembalinya sang kakak, maka jawaban yang paling tepat
adalah
(C) returns.

2
2. “Can we postpone the class meeting until Friday?”.
“I’m afraid not. I … basketball on Friday.” (UMPTN/2002/R-III/711)
A. would play
B. will have been playing
C. has been playing
D. always play
E. played
Pembahasan :
Dalam kalimat “I … basketball on Friday” subjek menyatakan kebiasaannya bermain
basket pada hari Jumat. Sehingga kalimat yang paling tepat untuk mengisi kata-kata
tersebut adalah (D) always play.
Jawaban: D

3. “Is no one living with your grandmother in that house?”


“No. And she … anyone do anything for her for years.” (SPMB/2003/R-III-312)
A. never had
B. does not have
C. has never had
D. never has
E. will never have
Pembahasan :
Dalam kalimat tersebut dikatakan bahwa sang nenek telah hidup sendirian selama
bertahun-tahun. Dari durasi yang ada dalam kalimat tersebut maka jawaban yang paling
tepat yakni (C) has never had (present perfect).
Jawaban: C

4. “Do you often go to the restaurant?”


“No. It … quite a long time since I went to a restaurant with my friends.” (SPMB/2004/R-
I/440)
A. has been
B. would be
C. is being
D. was
E. had been
Pembahasan :
Dalam kalimat tersebut, subjek mengatakan bahwa dia sudah cukup lama tidak pergi ke
restoran tesebut sejak terakhir kali dengan temannya sampai sekarang. Maka, digunakan
Present Perfect untuk melengkapi kalimat tersebut yakni (A) has been.
Jawaban: A

3
5. “Is the rescue team still looking for survivors of the plane crash?”
“Yes. They … the area for hours, but they haven’t found any yet.” (SPMB/2004/R-I/140)
A. searched
B. were searching
C. have been searching
D. will be searching
E. had searched
Pembahasan :
Dalam kalimat tersebut, tim penyelamat telah berusaha mencari korban yang selamat
dalam kecelakaan pesawat selama berjam-jam. Dan mereka masih terus mencari
keberadaan korban. Berdasarkan durasi yang terdapat dalam kalimat, maka jawaban
yang paling tepat yakni menggunakan Present Perfect Continous Tense atau (C) have been
searching.
Jawaban: C

6. “Have you already decided where to go for your holiday?”


“Certainly, we … to Italy.” (SPMB/2004/R-III/741)
A. go
B. have gone
C. will have gone
D. are going
E. will have been going
Pembahasan :
Kalimat tersebut menanyakan rencana liburan seseorang, yang mana orang tersebut
telah mempunyai tujuan yang pasti untuk berlibur. Maka jawaban yang paling tepat
untuk pertanyaan tersebut adalah (D) are going  to be + Verb-ing menyatakan niat dari
pelaku.
Jawaban: D

7. Every night the watchman turns on all the lights and … around the building every half
an hour. (UMPTN/1992/R-A/32)
A. walks
B. is walking
C. to be walking
D. walking
E. to walk
Pembahasan :
Dalam kalimat tersebut tedapat adverb of time ‘every’ yang menyatakan suatu aktivitas/

4
kebiasaan yang dilakukan setiap malam. Maka jawaban yang paling tepat untuk soal
tersebut dengan menggunakan Simple Present yakni kata (A) walks.
Jawaban: A

8. “My uncle’s office stays open until 12.00 on weekends.” The sentence means …. (UM-
UGM/2004/322)
A. my uncle works at 12.00 on weekends
B. my uncle’s office opens at 12.00 on weekends
C. my uncle doesn’t have to work on weekends
D. my uncle works every weekends
E. my uncle’s office closes at 12.00 on weekends
Pembahasan :
Pengertian yang paling tepat untuk kalimat tersebut adalah (E) karena kantor pamannya
tersebut hanya buka hingga pukul 12 pada akhir minggu.
Jawaban: E

9. “Why do you look so unhappy, Anton? What’s the


matter?” “My father … his job.” (UMPTN/1999/R-A/43)
A. has just lost
B. has been losing
C. losing
D. is losing
E. loses
Pembahasan :
Kejadian yang terjadi pada ayah Anton merupakan peristiwa yang baru saja terjadi
saat ini. Sehingga jawaban yang paling tepat dengan menggunakan Present Perfect
yakni (A) has just lost.
Jawaban: A

10. All these years, Ira’s family … in poverty. (UMPTN/1991/R-A/63)


A. lived
B. has been living
C. had lived
D. was living
E. lived
Pembahasan :
Karena kejadiannya telah berlangsung selama bertahun-tahun ini dan sampai sekarang
pun masih berlangsung, maka jawaban yang paling tepat dengan menggunakan
Present Perfect Continous yakni (B) has been living.
Jawaban: B

5
BAHASA InggRIS
Chapter 6 Past Tense

Past Tense adalah tenses yang digunakan untuk menyatakan kejadian yang berlangsung di
waktu lampau, baik yang terjadi dalam waktu yang spesifik, kejadian yang sedang
berlangsung di waktu lampau, maupun kejadian yang terjadi secara beurutan di waktu
lampau. Secara umum, Past Tense terbagi menjadi empat bagian, yaitu simple past, past
continuos, past perfect, dan past perfect continuous.

A. SIMPLE PAST
1. Rumus dasarnya: S + Verb 2 + Adverb of Time (in
Past) 2.. Digunakan untuk:
• Menyatakan kegiatan yang terjadi pada waktu tertentu di masa lampau.
Contoh:
He met Vivi yesterday.

• Menyatakan kebiasaan yang terjadi di masa lampau.


Contoh:
She always carried her umbrella on the last season.

• Adverb of time (penanda waktu) yang sering digunakan, yaitu yesterday,


last…, …ago, in…, from…to, this/that…

1
B. PAST CONTINUOUS
1. Rumus dasarnya: S + to be past + Verb-ing
2. Digunakan untuk:
• Menyatakan kegiatan yang sedang berlangsung di masa lampau.
Contoh:
Aries’s wife was cooking in the kitchen.

• Menyatakan kegiatan yang sedang berlangsung di masa lampau saat kejadian


lain terjadi/menyela.
Contoh:
The children were laughing when their father arrived home.

• Menyatakan kegiatan yang terjadi secara beriringan/bersamaan pada masa


lampau.
Contoh:
The people were watching the football match while Ronaldo was entering the field.

• Menyatakan keinginan di masa lampau, namun keinginan itu tidak dilaksanakan.


Contoh:
I was going to publish my book. (but I didn’t).

• Lebih menitikberatkan pada kejadian yang berlangsung, bukan keterangan


waktunya saat kejadian tersebut berlangsung.

C. PAST PERFECT
1. Rumus dasarnya: S + Had + Verb 3
2. Digunakan untuk:
• Menyatakan kegiatan yang terjadi sebelum kegiatan lain di masa lampau.
Contoh:
The sun had set down before I was ready to go.

• Menyatakan peristiwa yang terjadi selama periode tertentu hingga peristiwa


kedua terjadi di waktu lampau.
Contoh:
He had saved up for five years before he bought a car.

• Adverb of time yang umumnya digunakan, yaitu when, before, after, for a …
yang diikuti oleh clause (past tense).

2
D. PAST PERFECT CONTINUOUS
1. Rumus dasarnya: S + had been + Verb-ing
2. Digunakan untuk menyatakan suatu kegiatan/peristiwa yang dimulai di masa
lampau dan berlangsung terus sampai waktu itu atau baru saja berhenti sebelum
orang itu berbicara.
3. Hampir sama dengan Past Perfect, namun lebih menitikberatkan pada proses
berlangsungnya peristiwa.
Contoh:
I had been waiting for a bus for twenty minutes when I saw you.

CONTOH SOAL

1. “When did the guests come yesterday?”


“When she … cooking.” (UMPTN/2000/R-A/46)
A. had just finished
B. is finishing
C. was finished
D. has finished
E. finishes
Pembahasan :
Kalimat pertama menggunakan kata did yang menyatakan Past Tense, sedangkan kalimat
kedua menggunakan kata hubung when yang menyatakan Past Perfect. Jadi, pilihan A
menjadi jawaban yang paling tepat.
Jawaban: A

2. ASEAN Trade Ministers … the possibility of developing economic cooperation before


they attended the summit meeting last month. (UMPTN/2001/R-A/240)
A. discuss
B. discussed
C. had discussed
D. were discussing
E. would discuss
Pembahasan :
Kalimat di atas, menggunakan kata hubung before yang menyatakan Past Perfect
(kejadian yang berlangsung sebelum kejadian lain di masa lampau). Jadi, pilihan C
menjadi jawaban yang paling tepat.
Jawaban: C

3
3. “Last year I didn’t have to pay for my brother’s tuition fee.”
“That’s great. I heard he … a scholarship from a foreign university.”
A. receives
B. had received
C. was receiving
D. would receive
E. has received
Pembahasan :
Kalimat pertama menggunakan keterangan waktu last year yang menyatakan Past Tense.
Selain itu, kalimat tersebut menyatakan subjeknya (I) tidak perlu membayar SPP adiknya
tahun lalu yang berarti peristiwa/pembayaran tersebut telah dilakukan di masa lampau.
Jadi, pilihan B menjadi jawaban yang paling tepat dengan menggunakan Past Perfect
Tense.
Jawaban: B

4. “Your train was delayed, wasn’t it?”


“Yes, therefore, when I arrived, the driver picking me up … for more than an hour.”
A. waited
B. would wait
C. was waiting
D. would have waited
E. had been waiting
Pembahasan :
Pertanyaan pada kalimat pertama terdapat penanda Past Tense yang ditunjukkan dengan
for more than an hour. Karena terdapat durasi pada kalimat tersebut, pilhan E menjadi
jawaban yang paling tepat dengan menggunakan Past Continuous.
Jawaban: E

5. “Vina had just finished her big dinner when Jody came in, bringing her favorite durian.
” We may conclude that … “ (UM-UGM/2003/121)
A. Vina had durian for her dinner.
B. Vina and Jody had dinner together.
C. Vina would most unlikely have the durian for dinner.
D. Vina would have dinner after eating durian.
E. Vina must not have finished her dinner.
Pembahasan :
Soal tersebut memerlukan pemahaman dalam mencerna suatu kalimat. Kalimat
pertama menyatakan Vina baru saja menyelesaikan makan malam yang cukup

4
banyak. Oleh karena itu, Vina menjadi tidak menyukai makan malamnya. Alasannya,
buah kesukaannya (durian) justru datang saat dia sudah makan dan merasa kenyang.
Jawaban: C

6. “You were supposed to be here ten minutes ago. Where were


you?” “Yes. I … for a place to park.” (SPMB/2004/R-2/240)
A. am looking
B. looked
C. have looked
D. look
E. was looking
Pembahasan :
Kalimat pertama menyatakan teman subjek menanyakan keberadaan subjek karena
seharusnya dia sudah berada di tempat tersebut, sepuluh menit yang lalu. Subjek
mengatakan dia sedang mencari tempat parkir selama sepuluh menit tersebut. Karena
kejadian tersebut sedang berlangsung di masa lampau, pilihan E menjadi jawaban
yang paling tepat dengan menggunakan Past Continuous.
Jawaban: E

7. “I was the first to submit my paper yesterday.”


“You’re wrong. I … mine before you did.” (SPMB/2006/R-1/111)
A. submit
B. have submitted
C. was submitting
D. would sumit
E. had sumitted
Pembahasan :
Kalimat pertama menggunakan Past Tense yang ditunjukkan to be was.
Kalimat kedua terdapat kata penghubung (conjunction) before yang merupakan
penanda Past Perfect. Jadi, pilihan E menjadi jawaban yang paling tepat.
Jawaban: E

8. I … all over the place for my dictionary when I suddenly remembered that my sister
had borrowed it. (UMPTN/2000/R-B/140)
A. have looked
B. was looking
C. looked
D. am looking
E. have been looking

5
Pembahasan :
Kalimat tersebut menggunakan penanda Past Tense yang ditunjukkan kata remembered.
Selanjutnya, kalimat tersebut merupakan Past Continuous karena menyatakan
kegiatan yang sedang berlangsung di masa lampau yakni ‘dia sedang mencari
kamusnya’ ketika kegiatan lain menyela yakni ‘dia teringat bahwa kakaknya telah
meminjam kamus tersebut.’
Jawaban: B

9. He put out the light. Then, he locked the door.


The above sentences can be joined as follow … (UMPTN/1190/R-A/53)
A. When he locked the door, he put out the light.
B. He would put out the light if he locked the door.
C. He had put out the light before he locked the door.
D. He put out the light after he locked the door.
E. While he was locking the door, he put out the light.
Pembahasan :
Soal tersebut termasuk soal pemahaman. Kedua kalimat di atas, terjadi dua buah
peristiwa yang terjadi secara berurutan di masa lampau. Dalam kalimat di atas,
pelaku (subject) mematikan lampu terlebih dahulu sebelum dia mengunci pintu. Jadi,
jawaban yang paling tepat dengan menggunakan Past Perfect Tense.
Jawaban: C

6
BAHASA InggRIS
Chapter 7 Conjunctions

Conjunction (kata penghubung) biasanya digunakan untuk menyambungkan kata (a book


and a pen), phrases (frasa) atau sekumpulan kata (over the table but near the clock), atau
menghubungkan clause atau kalimat (Amir is happy, but Susi is sad). Secara umum, conjuctions
terbagi menjadi empat bagian, Coordinate Conjunctions, Correlative Conjunctions, Subordinate
Conjunctions, dan Conjunctive Adverb.

A. COORDINATE CONJUNCTIONS
Menghubungkan compound sentence (for, and nor, but, or, yet, dan so).
a. Mr Ahmad planned to go abroad, and he would take Vivi with him.
b. I miss you but I hate you.
c. Please send me a letter or call me as soon as you arrive there.
d. Tom doesn’t like durian nor watermelon.
e. I can’t go, for I don’t have much time to go.
f. Come closer, so I can see you clearly.
g. My boss is talkative, yet he is a kind person.

B. CORRELATIVE CONJUNCTIONS
Kata penghubung yang harus berpasangan.
a. Irma is not only smart but also pretty.
b. Both Leo and Emily are in Bogor now.

1
c. He can neither read nor write.
d. You can travel to Bali either by bus or by plane.

C. SUBORDINATE CONJUNCTIONS
Kata penghubung yang menyambungkan klausa tidak setara. Umumnya, digunakan
dalam complex sentence.
a. After he had written the letter, he posted it.
b. As soon as the teacher came, the lesson was started.
c. She walked as if she were a model.
d. I study English because I want to go abroad.
e. Unless Mr. Jimmy asks, we can collect this paper to him.
f. Since I am sick, I can’t work.
g. I save my money so that I can buy a new car.
h. Though Jane was sick, she still came to the class.
i. Ali is lazy whereas his sister is very diligent.
j. In spite of the rain, I go to school.
k. You can go on in case I don’t come here.
l. If you study hard, you will get a good score. On the other hand, you will fail if you don’t.
m. As he was leaving the house, someone came to his house.

D. CONJUNCTIVE ADVERB
Kata penghubung yang berfungsi menyambungkan klausa atau kalimat.
Contoh:
thus (dengan demikian); moreover dan furthermore (lagipula); besides dan in addition (di
samping itu); therefore, accordingly, hence, dan as a result (oleh karena itu); nevertheless,
however, dan yet (namun).
a. I’d like to go; however, I don’t have any money.
b. You must have enough rest; otherwise, you will get sick.
c. Joanna is so beautiful. Thus, she is popular among boys.
d. Ajeng is clever. In addition, she is also very rich.
e. My brothers are very genius. Moreover, they also polite and humble.

2
CONTOH SOAL

1. … the great inconvenience of travelling, many television reporters went to the


flooded area for their reports. (SPMB/2002/R-2)
A. despite
B. in addition to
C. although
D. in case of
E. because of
Pembahasan :
Awal kalimat tersebut merupakan frasa. Ini karena hanya terdapat satu subjek dalam
kalimat. Kalimat tersebut juga menyatakan suatu pertentangan makna antara kalimat
pertama yang bermakna negatif dengan kalimat kedua yang bermakna positif Jadi, kata
penghubung yang paling tepat untuk melengkapinya adalah despite (pilihan A).
Jawaban: A

2. Smoking can be the cause of many illnesses and respiratory disorders … it may harm
non- smokers. (UMPTN/2000/R-A & UMPTN/2001/R-B/R-C)
A. consequently
B. however
C. in addition
D. nevertheless
E. otherwise
Pembahasan :
Kata however dan nevertheless memiliki makna yang sama, yakni namun
demikian. Jadi, jawaban tersebut kurang tepat. Demikian juga dengan
pilihan A dan E, consequently menyatakan akibat dan otherwise menyatakan
syarat. Jadi, jawaban yang paling tepat pada pilihan C, in addition.
Jawaban: C

3. … three people have died of Cholera, the old people of the village refused to get
an injection against it. (UMPTN/2000/R-B)
A. although
B. however
C. in spite of
D. since
E. despite

3
Pembahasan :
In spite of dan despite memiliki makna dan fungsi yang sama, yakni bermakna
walaupun yang sama-sama harus diikuti oleh frasa. Demikian juga dengan kedua kata
lainnya, however (namun demikian) dan since (sejak atau karena). Kalimat tersebut
merupakan klausa. Jadi, although pada pilihan A menjadi jawaban yang paling benar.
Jawaban: A

4. There was nothing for dinner, … we had to go to a grocery store. (SPMB/2004/R-I)


A. otherwise
B. moreover
C. besides
D. however
E. therefore
Pembahasan :
Kalimat tersebut menyatakan sebab-akibat yang ditunjukkan there was
nothing for dinner (tidak ada apapun untuk makan malam). Jadi, kata yang
dapat melengkapi kalimat selanjutnya, yakni therefore (oleh karena itu).
Sementara pilihan lainnya kurang tepat, yakni otherwise (syarat), moreover
(lagipula), besides (di samping itu), dan however (namun demikian).
Jawaban: E

5. “Did he say something to the police”


“Well, … he admitted breaking into the car, he denied stealing the tape.” (SPMB/2002/
R-I)
A. eventhough
B. because
C. meanwhile
D. when
E. as soon as
Pembahasan :
Kalimat tersebut menyatakan suatu pertentangan karena bermakna dia terbukti merusak
bagian dalam mobil, dia menyangkal dirinya mencuri radio. Jadi, jawaban yang paling
tepat untuk mengisi bagian rumpang tersebut, yaitu pilihan A eventhough (meskipun
demikian) .
Jawaban: A

6. The hotel provides good facilities, yet … (SPMB/2002/R-II)


A. The occupancy rate is very low.
B. Foreign tourists like to stay there.
C. It is located on the beautiful beach.

4
D. On-the-job-training is provided for the employees.
E. It is advertised on the yellow pages.
Pembahasan :
Kalimattersebutmerupakankalimat yang salingbertentangandenganadanyakatahubung
yet(namunatautetapi).Katayetinimembuatbagianyangrumpang harusdilengkapidengan
kalimat yang bermakna bertentangan dengan kalimat sebelumnya (hotel itu
menyediakan fasilitas yang baik, namun...). Oleh karena itu, pilihan A menjadi jawaban
yang paling tepat.
Jawaban: A

7. Our dental clinic should be provided with more modern equipments, otherwise …
(SPMB/2003/R-III)
A. Patients would be eager to come.
B. It would make patients comfortable.
C. Patients would not be cured.
D. It would become more popular.
E. No patients would come.
Pembahasan :
Kalimattersebutdiikutikatapenghubungyangmenyatakansyarat,yaituotherwise(jikatidak).
Olehkarenaitu,kalimatselanjutnyaharusdilengkapidengankalimat yangmenyatakanakibat
untuk melengkapi kalimat sebelumnya yang bermakna sebab (klinik gigi kita
seharusnya dilengkapi dengan peralatan modern). Jadi, pilihan E menjadi jawaban yang
paling tepat.
Jawaban: E

8. This project is a huge construction project which needs the use of high technology; it
is therefore most likely that the government will have an established foreign company to
do the job.
From the above sentence we may conclude that … (SPMB/2004/R-III)
A. The government seems to be incapable of doing the job.
B. The construction project should be completed within a short time.
C. There are not enough technicians to be involved in the project.
D. The government may not have the necessary financial resources.
E. The project is performed based on a 40-60 share.
Pembahasan :
Secara implicit (tersirat), dapat diambil kemungkinan terbesar bahwa akhirnya,
pemerintah memutuskan untuk menunjuk perusahaan asing untuk mengerjakan
proyek tersebut disebabkan pemerintah dirasa tidak mampu mengerjakan proyek yang
menggunakan teknologi tinggi. Jadi, jawaban yang paling memungkinkan, yaitu pilihan
A.
Jawaban: A

5
9. People like to live in the peaceful world, nevertheless … (UMPTN/1998/R-C)
A. Every conflict and disagreement should be avoided.
B. They try very hard to live in harmony.
C. Coordination and cooperation are being conducted.
D. They come together to solve global problems.
E. There are still wars among nations.
Pembahasan :
Kata penghubung yang terdapat pada kalimat tersebut, yakni nevertheless (namun).
Kata penghubung ini menunjukkan suatu pertentangan dari kalimat sebelumnya yang
bermakna positif (Masyarakat senang hidup damai di dunia, namun ...). Oleh karena itu,
kalimat selanjutnya harus bermakna negatif. Jawaban yang paling tepat, yaitu pilihan E.
Jawaban: E

6
BAHASA InggRIS
Chapter 8 Passive Voice

Passive voice (kalimat pasif) adalah suatu kalimat yang subjeknya dikenai tindakan, perbuatan,
atau pekerjaan. Dalam kalimat aktif, subjek yang melakukan suatu perbuatan. Dalam kalimat
pasif, objek yang akan menjadi subjek, namun terlebih dahulu disisipi prefix (imbuhan)
yang bermakna di- atau ter-.

Seperti diketahui, ada dua macam verb (kata kerja), yaitu Transitive (memerlukan objek) dan
Intransitive (tidak memerlukan objek).
a. Transitive: buy, drink, eat, make, dan etc.
b. Intransitive: go, sleep, come, run, dan etc.
Hanya kalimat yang mengandung Transitive Verb yang dapat diubah menjadi kalimat pasif
karena objek kalimat aktif akan menjadi subjek dalam kalimat pasif.

Kalimat pasif digunakan jika kita menganggap tidak perlu mengungkapkan si pelaku. Jadi,
subjek dalam kalimat aktif bisa disebutkan ataupun dihilangkan setelah diubah ke dalam
kalimat pasif.

Secara umum, penggunaan dan pola Passive Voice terbagi menjadi lima bagian:
1. Present Tense ( S + to be present + Verb 3)
Contoh:
A snake bites Steve.  Steve is bitten by a snake.

1
2. Past Tense ( S + to be past + Verb 3)
Contoh:
Mother cooked some eggs.  Some eggs were cooked by mother.

3. Continuous Tense ( S + to be + being + Verb 3)


Contoh:
The children are playing football.  Football is being played by them.

4. Perfect Tense ( S + have/has + been + Verb 3)


Contoh:
He has written several letters to his girlfriend.  Several letters have been written by him to

5. Modal ( S + modal + be + Verb 3)


Contoh:
I will buy a new house.  A new house will be bought by me.

Note:
Subjek pada kalimat aktif menjadi objek pada kalimat pasif.
Tenses pada kalimat pasif mengikuti bentuk kalimat aktifnya.
V3 pada kalimat pasif dibentuk verb pada kalimat aktif.
Kalimat pasif digunakan untuk menonjolkan hasil tindakan daripada pelaku tindakan itu sendiri.
Hanya biasa digunakan pada kalimat yang memiliki objek (transitive verb).

CONTOH SOAL

1. When the bus broke down in the middle of the dessert, we … that we would have to
finish the journey to our destination on foot. (SPMB/2005/R-II/270)
A. were telling
B. have told
C. were told
D. have been told
E. had told
Pembahasan :
Pola dasar dari kalimat pasif, yaitu S + to be + Verb 3. Jadi, jawaban yang paling tepat
untuk kalimat tersebut ialah were told, pasangan untuk subjek we ialah were.
Jawaban: C

2
2. “How old is this mosque?”
“Well, it … it in 1870.” (UMPTN/1993/R-B-53)
A. built
B. to be built
C. was built
D. had built
E. had been building
Pembahasan :
Kalimat tersebut terdapat penanda Simple Past pada kata in 1870. Jadi, pilihan C menjadi
jawaban yang paling tepat untuk kalimat pasif di atas.
Jawaban: C

3. “Last night a thief broke into my brother’s house.”


“Really? What … from the house?” (SPMB/2002/R-II/310)
A. he took
B. it was taken
C. was being taken
D. was taking
E. was taken
Pembahasan :
Dalam kalimat tersebut, kejadianya pada kemarin malam (last night) menyatakan
kejadian yang terjadi di masa lampau. Di kalimat selanjutnya ditanyakan apa yang
telah diambil dari rumah tersebut. Jadi, pilihan E menjadi jawaban yang paling tepat.
Jawaban: E

4. Jupiter’s four moons … through a telescope by Galileo. (UM-UGM/2004/322)


A. were first viewed
B. first viewed
C. had been first viewed
D. were being first reviewed
E. first being reviewed
Pembahasan :
Planet Jupiter memiliki empat buah satelit (bulan)  plural. Sementara itu, Galileo
merupakan tokoh hebat pada zamannya yang melakukan pengamatan melalui
teleskop. Karena kejadian tersebut berlangsung di masa lampau, kalimat tersebut
menggunakan pola Past Tense dalam kalimat pasif. Jadi, pilihan A menjadi jawaban
yang paling tepat.
Jawaban: A

3
5. The Amazon Valley is extremely important to the ecology of the earth. Forty percent of
the world’s oxygen … there. (UMPTN/2000//R-C/12)
A. are being produced
B. are produced
C. is being produced
D. is produced
E. was produced
Pembahasan :
Kalimat tersebut menggunakan pola Present Tense yang ditandai
oleh penggunaan to be is. Forty percent of the world’s oxygen …
 jika terdapat satuan maka kita hitung sebagai satuan (singular).
Jawaban: D

6. “When did the accident happen?”


“When the goods … from the truck.” (UMPTN/1993/R-C/23)
A. were unloading
B. were being unloaded
C. being unloaded
D. they were unloading
E. unloaded
Pembahasan :
Kalimat tersebut menanyakan ‘kapan kecelakaan itu terjadi?’ dan jawaban yang paling
tepat adalah ‘ketika barang-barang tersebut dimasukkan’  menyatakan kejadian
yang sedang berlangsung pada saat itu dengan menggunakan Past Continuous.
Jawaban: B

7. “When will the debate contest take place this year?”


I’ll let you know later as it ….”
(SPMB/2006/R-III/710)
A. has not scheduled yet
B. has been scheduled
C. has to schedule
D. has been scheduling
E. has not been scheduled yet
Pembahasan :
Kalimat tersebut menanyakan, “Kapan kontes debat akan diselenggarakan tahun
ini?” Namun, hal tersebut belum dapat dipastikan karena subjek pada kalimat kedua
mengatakan, “Dia akan memberi kabar secepatnya karena jadwalnya belum dijadwalkan.”
Jawaban: E

4
8. “Bambang looks very happy today.”
“Don’t you know that he … to General Manager?” (UMPTN/1997/R-A/31)
A. has promoted
B. to be promoted
C. being promoted
D. has to promote
E. has been promoted
Pembahasan :
Dalam kalimat tersebut, Bambang terlihat sangat bahagia karena
dia telah dipromosikan (Perfect Tense  been + V3) ke General
Manager. Jadi, pilihan E menjadi jawaban yang paling
tepat.
Jawaban: E

5
BAHASA InggRIS
Chapter 9 Modals

Auxiliary adalah kata kerja bantu. Modal adalah bagian dari auxiliary yang merupakan
sekelompok kata kerja bantu yang memberikan arti tambahan dalam suatu kalimat. Yang
harus diingat ialah modal selamanya diikuti kata kerja bentuk infinitive (Verb 1). Secara umum,
terdapat dua macam Modal yang kita perlu ketahui, yaitu Modal Auxiliary dan Modal Perfect.

A. MODAL AUXILIARY
Selalu diikuti oleh infinitive (Verb 1).
Contoh:
1. May: Mrs. Lusi may become the best teacher this term.
(possibility) May I borrow your pen, please? (permission)
2. Can: I can swim well. (ability)
It can rain tomorrow. (possibility)
3. Shall: Shall I open the window? (asking)
You shall go to the doctor.
(advice)
4. Will: He will take some courses. (going to happen)
5. Must: Students must study hard. (necessity)
You look pale. You must be sick. (possibility)
6. Used to: I used to cry when my mother left me alone. (habitual actions)

1
B. MODAL PERFECT
Selalu diikuti oleh Verb 3 dan menyatakan masa lampau.
Contoh:
1. Could have + Verb 3
Menyatakan hal yang sebenarnya biasa saja dilakukan, tetapi tidak dilakukan di
masa lampau.
Contoh:
She could have asked her brother to lift up the box. (She lifted the box herself)

2. Might have + Verb 3


Menyatakan suatu kemungkinan/praduga atas sesuatu yang telah terjadi di masa
lampau.
Contoh:
His room was still very dirty, he might have forgotten to clean it.

3. Should have + Verb 3


Menyatakan suatu peristiwa yang contrary (berlawanan) dengan fakta.
Contoh:
John should have gone to his office this morning. (He didn’t go)

4. Must have + Verb 3


Menyatakan suatu kesimpulan dari apa yang telah terjadi di masa lampau. Harus
diingat bahwa menyatakan keharusan di masa lampau hanya boleh menggunakan
had to atau should have + Verb 3.
Contoh:
The grass is wet. It must have rained last night.

CONTOH SOAL

1. “What should the country do to maintain self-sufficient in rice?”


“It … double its rice production.” (SPMB/2003/R-III/312)
A. could
B. must
C. had to
D. would
E. will

2
Pembahasan :
Kalimattersebutmenanyakan,“Apayangseharusnyadilakukanuntukmenjagaswasembada
beras?” Kalimat tersebut menggunakan pertanyaan dalam bentuk Present do. Jadi, pilihan
B menjadi jawaban yang paling tepat karena menggunakan bentuk Present Tense (must).
Jawaban: B
2. “I tried to call Tini on her mobile phone yesterday, but I only got her mailbox”
“She … to bring her phone. She often does that.” (SPMB/2004/R-III/541)
A. had to forget
B. should have forgotten
C. must forget
D. must have forgotten
E. ought to forget
Pembahasan :
Dalam kalimat tersebut, subjek pertama berusaha menghubungi Tini beberapa kali, namun
hanya pesan suara yang dia dapat. Pada kalimat selanjutnya, subjek kedua memberikan
kesimpulan bahwa Tini pasti lupa membawa ponselnya karena hal tersebut sering terjadi.
Jawaban: D

3. “My car was stolen from the garage and I don’t know if I can buy another one”.
“You … it.” (SPMB/2005/R-II/270)
A. had to insure
B. should have insured
C. will have insured
D. ought to insure
E. must have insured
Pembahasan :
Dalam kalimat tersebut, mobil subjek telah dicuri dari garasi rumahnya dan
dia sendiri pun tidak yakin dirinya mampu membeli mobil lain ke depannya.
Lawan bicaranya pun menyarankan agar dia, seharusnya, mengasuransikan
mobilnya. Jadi, modal yang paling tepat untuk memberikan saran, yaitu pilihan B.
Jawaban: B

4. “I need more data about this


topic.” “Rawina might be able to
help you”. Means …
(UMPTN/1990/R-A/53)
A. Perhaps Rawina can help you.
B. Rawina can help you right now.
C. I’m sure Rawina will be able to help you.
D. Rawina will certainly be able to help you.
E. Rawina was able to help you.

3
Pembahasan :
Pada kalimat tersebut, modal yang digunakan adalah might yang berasal dari kata dasar
may yang menyatakan kemungkinan (possibility). Jadi, tidak dapat dikataka bahwa
Rawina pasti dapat menolong karena itu masih merupakan sebuah kemungkinan.
Jawaban: A
5. “I’m glad Adi’s academic performance has greatly improved this
semester.” “I bet, he … hard for that.” (SPMB/2006/R-III/510)
A. should have worked
B. has to work
C. must have worked
D. would work
E. ought to have work
Pembahasan :
Prestasi Adi telah meningkat pesat pada semester ini. Melihat hasil yang
diraih oleh Adi pada saat ini, bisa ditarik kesimpulan bahwa dia pasti sudah
bekerja dan berusaha keras untuk mencapai semuanya. Jadi, jawaban yang
paling tepat ialah must have worked karena melihat hasil yang di dapat.
Jawaban: C

6. The curtains are dirty. We … at the dry cleaner. (UMPTN/1991/R-A/63)


A. must have washed them
B. have washed them
C. should have them washed
D. are washing them
E. washing them
Pembahasan :
Karena gorden kotor, mereka harus mencucinya di laundry.
Jadi, pilihan A menjadi jawaban yang paling tepat..
Jawaban: A

7. “I ran through the red light and I got fined by the traffic
policeman.” “Oh, you … more careful.” (UMPTN/2002/R-III/710)
A. must have been
B. could be
C. might have been
D. maybe
E. should have been
Pembahasan :
Mengacu pada kasus yang dihadapi akibat ketidaksabaran subjek pertama, subjek
kedua memberikan saran agar subjek pertama lebih berhati-hati lagi ke depannya.
Jawaban: E

4
BAHASA InggRIS
Chapter 10 Causative

Causative adalah kata kerja yang berarti menyuruh/meminta seseorang melakukan suatu
kegiatan tertentu atau dengan kata lain menyuruh/meminta sesuatu dilakukan. Pada intinya,
subjek dalam kalimat causative tidak pernah melakukan pekerjaan tersebut dengan tangannya
sendiri.

Secara garis besar, Causative Verbs (CV) terbagi menjadi dua kelompok:
Have/has/had
Make/makes/made + Verb 1 Get/gets/got + to Verb 1*
Let/lets/let

Selanjutnya, pola kalimat terbagi menjadi dua bagian, yakni:


1. Active : S + CV + Person + Verb 1
2. Passive : S + CV + Thing + Verb 3

Note:
(*) hanya berlaku untuk kalimat aktif
Have/has/had  menyuruh/meminta
Make/makes/made  memaksa
Let/lets/let  membiarkan
Get/gets/got  menyuruh/meminta

1
Contoh:
She has the boy sweep the floor. (active)
She has the floor swept by the boy. (passive)
The teacher made us bring some cookies. (active)
The teacher made some cookies brought by us. (passive)
My mother let me cut my hair. (active)
My mother let my hair cut. (passive)
Mia gets Aries to fix the roof. (active)
Mia gets the roof fixed by Aries. (passive)

CONTOH SOAL

1. Since her handwriting is very bad, we will have her … her composition. (UMPTN/1990/
R-A/53)
A. types
B. typed
C. typing
D. type
E. to type
Pembahasan :
Dalam kalimat tersebut, setelah Causative Verb have, kata selanjutnya
ialah her yang menunjukkan seseorang (perempuan). Oleh karena itu,
kalimat tersebut termasuk kalimat aktif yang subjeknya diikuti oleh Verb1.
Jawaban: D

2. It is impossible for us to translate the whole book within two weeks. So we ….


(UMPTN/2010/R-A/240)
A. have to translate it by ourselves
B. have some assistants help us
C. have translated the whole book
D. have been asked to translate it
E. have given all the transaction to the assistants
Pembahasan :
Karena kami (subjek) merasa tidak mampu untuk menerjemahkan keseluruhan
isi buku dalam waktu dua minggu, kami memutuskan untuk menyuruh beberapa
asisten untuk membantu kami. Jadi, pilihan B menjadi jawaban yang paling tepat.
Jawaban: B

2
3. “When do you want to have the flowers … to your house?”
A. to deliver
B. delivering
C. be delivered
D. delivered
E. deliver
Pembahasan :
Kalimat tersebut menanyakan, “Kapan kiranya bunga-bunga
tersebut dapat dikirim ke rumah?” Sesuai dengan pola Causative
Verb (CV), setelah CV itu benda maka akan diikuti oleh V3.
Jawaban: D

4. “After finished my article, I had it edited.”


Means … (UMPTN/1998/R-C/42)
A. I had edited my article.
B. My article would be edited.
C. Somebody edited my article.
D. I was asked to edit my article.
E. I had to edit my article myself.
Pembahasan :
Kalimat I had it edited berarti saya menyuruh seseorang untuk mengedit artikel yang
sudah
sayabuatsebelumnya.Dengankatalain,seseorangmengedityangsayabuatdanbukansaya.
Jawaban: C

5. “We need to replace the glass in the picture.”


“We can go to the shop and … now.” (UM-UGM/2003/121)
A. it’s being done
B. have it done
C. have done
D. it’s done
E. do it
Pembahasan :
Ketika kita pergi ke toko untuk memperbaiki kaca pada pigura tersebut,
bukan kita yang akan memperbaiki pigura melainkan kita menyuruh orang
lain untuk melakukannya. Jadi, pilihan B menjadi jawaban yang paling tepat.
Jawaban: B

3
BAHASA InggRIS
Chapter 11 If Conditional

Jika kita bicara tentang If Clause atau Conditional Clause maka kita berbicara tentang
kalimat pengandaian yang terdiri dari dua bagian, yaitu Main Clause dan If Clause. Secara
umum, If Conditional terdiri dari tiga bagian, yaitu Future Possible, Present Unreal, dan Past
Unreal.

A. TIPE 1 (FUTURE POSSIBLE)


Menyatakan sesuatu yang mungkin terjadi di masa mendatang ataupun sekarang,
asalkan syaratnya terpenuhi. Future Possible merupakan satu-satunya tIpe dari If
Conditional yang memiliki kemungkinan untuk terwujud/terjadi di masa mendatang.
a. Rumusnya: If + Simple Present, Simple Future
b. Contoh:
If I have a lot of money, I will go around the world. (I may go around the world someday if I ha

B. TIPE 2 (PRESENT UNREAL)


Menyatakan sesuatu yang bertentangan dengan sesuatu yang terjadi saat ini. Adapun
peristiwa tersebut terjadi di masa lampau sehingga yang tersisa di masa sekarang
hanya penyesalan.

1
a. Rumusnya: If + Simple Past, Past Future
b. Contoh:
If he came to the party, I would dance with him all night long. (The fact is ‘He doesn’t c

c. Dalam Conditional tipe 2 ini, kita bisa menggunakan to be were untuk semua subjek
karena kalimat pengandaian.
Contoh:
If I were Doraemon, I would fly with Baling-baling Bambu. (I’m not Doraemon so I can’t fly)

3. TIPE 3 (PAST UNREAL)


Menyatakan sesuatu yang bertentangan dengan keadaan atau apa yang terjadi di masa
lampau.
a. Rumusnya: If + Past Perfect, Past Future Perfect
b. Contoh:
If he had told me the truth, we wouldn’t have broken. (He lied to me about the truth. T

c. Pada Conditional tipe 3 ini bisa berlaku pola Inversi (penghilangan kata ‘If’) dan
memindahkan had ke depan kalimat.
Contoh:
If we had known that you were there, we would have written you some letters.  Had we

CONTOH SOAL

1. Had I realized that Tim was a bad driver, I … my car. (UMPTN/1999/R-B/12)


A. would not lend him
B. did not lend him
C. will not lend him
D. had not lend him
E. would not have him

2
Pembahasan :
Kalimat tersebut merupakan Conditional tipe 3 yang menggunakan pola Inversi (If
dihilangkan). Pada tipe 3, sudah diketahui bahwa If Clause-nya menggunakan Past Perfect
TensemakapasangannyaPastFuturetense.Jadi, pilihanEmenjadi jawabanyangpalingtepat.
Jawaban
:E
2. If I don’t come to the class today, the teacher will have me write the poem ten times. The
underlined words means … (UMPTN/1992/R-C/33)
A. The teacher has written the poem ten times.
B. The teacher will write the poem for me ten times.
C. I will have to write the poem ten times.
D. I have written the poem ten times.
E. I will ask the teacher to write the poem ten times.
Pembahasan :
Kalimat yang digarisbawahi pada soal tersebut merupakan Simple Future yang artinya
berpasangandenganIf+SimplePresentdanmerupakanConditionaltipe1.Padatipe1,masih
terdapatkemungkinanuntukterjadidimasadepan, asalkanpersyaratannyaterpenuhi. Jadi,
subjek akan diharuskan menulis puisi sebanyak sepuluh kali jika dia tidak masuk hari itu.
Jawaban: C

3. If the car were in better condition, we would make a long trip now. It may be concluded
that we. . .(UMPTN/1993/R-C/23)
A. have to stay at home
B. are enjoying the trip
C. prefer staying at home
D. enjoyed the trip
E. stayed at home
Pembahasan :
KalimattersebutmenggunakanpolaConditionaltipe2,If+SimplePastberpasangandengan
Past Future. Berdasarkan makna dari kalimat tersebut, kita dapat simpulkan saat ini,
mereka tidak jadi menikmati perjalanan karena kondisi mobil yang tidak memungkinkan,
sehingga
merekaterpaksaharustinggaldirumahsaja.Jadi,pilihanAmenjadijawabanyangpalingtepat.
Jawaban: A

3
BAHASA InggRIS
Chapter 12 Subjunctive

Subjunctive adalah suatu ungkapan atau pengharapan yang menyatakan bahwa kita
menyesali sesuatu yang tidak sesuai dengan yang kita inginkan (bertentangan dengan
faktanya). Kalimat subjunctive menggunakan penanda pengandaian, yaitu wish (berharap),
if only (seandainya), as if dan as though (seolah-olah), serta would rather (lebih suka). Ada
beberapa poin yang perlu diingat tentang subjunctive, yaitu:
1. Selalu dalam bentuk Past (lampau)
2. Wish vs Reality
3. Semua subjek menggunakan were
4. Pola dasarnya:
Past  Present unreal
Past Perfect  Past unreal
Subjunctive terbagi menjadi empat bagian, yaitu wish, if only, as if/as though, dan would rather.

A. WISH
a. I wish I were a pilot. (I am not a pilot)
b. I wished I had bought a new car. (I didn’t buy a new car)

B. IF ONLY
a. If only I took the course. (I don’t take the course)
b. If only it hadn’t rained last night. (It rained last night)

4
C. AS IF/ AS THOUGH
a. She talks as if she knew everything. (She doesn’t know everything)
b. The man acted as though he had had the company. (He didn’t have it)

D. WOULD RATHER
a. I would rather they lived in Yogya. (they don’t live in Yogya)
b. I would rather you had finished doing the assignments. (You didn’t finish doing the
assignment yet)

CONTOH SOAL

1. I am sorry I can’t go to the airport to see your mother off. I wish I … to work overtime
tonight. (UMPTN/1996/R-B/12)
A. wouldn’t have
B. haven’t had
C. won’t have
D. hadn’t had
E. don’t have
Pembahasan :
Kalimat tersebut memiliki pengandaian dengan menggunakan Present Tense (Present
Unreal) untuk menyatakan hal yang sebenarnya bertentangan dengan dia inginkan.
Karena kalimat tersebut menggunakan subjunctive wish dalam bentuk present, kata
yang tepat untuk mengisi bagian tersebut ialah bentuk Past Tense (wouldn’t have).
Jawaban: A

2. I am sure he is not the man in charge of the sales department. But now, he acts …
(UMPTN/1991/R-A/63)
A. Acts as if he is the sales manager.
B. Is acting as if he would be the sales manager.
C. Would have acted as if he had been the sales manager.
D. Would act as ifhe was the sales manager.
E. Acts as if he were the sales manager.
Pembahasan :
Kalimat tersebut menyatakan, “Saya yakin dia bukanlah orang yang bertanggung
jawab terhadap departemen penjualan. Namun, dia hanya bertingkah seolah-

2
olah dia itu manajer penjualan.” Karena kalimat tersebut menggunakan pola Simple
Present untuk menggambarkan kenyataan yang ada, pasangannya ialah Past Tense.
Jawaban: E

3. ‘Problems of transportation have caused donated food and clothing for the tsunami
victims to be pilled up at the airport.’ I wish … (SPMB/2002/R-II/310)
A. There are more volunteers to help.
B. The victims should not have worried.
C. The government can ask for help.
D. The problem will be solved immediately.
E. There were more helicopters available.
Pembahasan :
Kalimat tersebut merupakan soal pemahaman terhadap bacaan. Pada kalimat
tersebut, permasalahan pokok ialah distribusi pangan dan pakaian untuk para korban
tsunami. Jadi, paling esensial dilakukan ialah menyediakan lebih banyak
helikopter untuk mempermudah distribusi barang-barang kebutuhan para
korban bencana alam.
Jawaban: E

4. “You look tired. Why don’t you take some rest?”


“I wish I … to work overtime tonight.” (UMPTN/1990/R-B/13)
A. i would have able
B. i can
C. i will be able
D. i couldn’t have
E. i had been able
Pembahasan :
Subjek berharap bahwa dirinya tidak harus bekerja lembur lagi malam
itu karena dia sudah terlihat sangat lelah. Karena kalimat penuturnya
menggunakan pola Present Unreal, kalimat pengandaiannya ialah Past Tense.
Jawaban: D

3
BAHASA InggRIS
Chapter 13 Concord

Concord adalah kesesuaian (agreement) antara subyek (noun) dengan kata kerja (verb)
atau kata kerja bantu (auxiliary) dalam suatu kalimat. Selain itu, concord juga merupakan
kesesuaian antara satu kata dengan kata lainnya (word agreement). Dengan kata lain, jika
subyeknya tunggal (singular) maka predikatnya juga akan menjadi singular dan begitu pula
sebaliknya.

Contoh:

The boy reading in the library is my brother.


The boys reading in the library are my brothers.

Terdapat beberapa aturan dalam Concord:


1. Jika subject mempunyai dua benda (nouns) yang dihubungkan dengan preposisi in,
on, at, dan of, verbnya bergantung pada benda di depan preposisinya.
Contoh:
The thick dictionary on the table is very useful for me.

2. Jika subjek mempunyai dua kata benda yang diapit oleh dua buah koma maupun kata-
kata, seperti together with, accompanied by, as well as, dan along with, tergantung
pada kata benda yang ada di depannya.

4
Contoh:
The governor, together with his wife and children, is having a journey to Raja Ampat Papua.

3. Jika subjek menggunakan artikel, tergantung jenis definite article (the  dianggap
singular) ataukah indefinite article (a  dianggap plural).
Contoh:
The number of days in a week is seven.
A number of students are going to have a picnic.

4. Kata every, each, each of (diikuti plural) namun dianggap singular.


Contoh:
Each girl in the room has a beautiful bag.

5. Penggunaan Gerund dan kata benda yang bersifat abstrak diikuti predikat yang
berbentuk tunggal (singular).
Contoh:
Jogging is my favorite sport.

6. Jika subjek menggunakan benda-benda yang berbentuk jamak atau sepasang,


predikatnya akan menjadi jamak pula.
Contoh:
My new socks are made of wool.

7. Jika subjek merupakan benda yang tidak bisa dihitung (uncountable noun), dianggap
singular.
Contoh:
The sugar is sweet.

8. Nama cabang ilmu maupun mata pelajaran dianggap singular.


Contoh:
Mathematics is the most difficult lesson for many students.

2
9. Pronoun (kata ganti) dengan antecedent (kata atau bagian kalimat yang mendahului)
harus sesuai.
Contoh:
All mothers in the world love their children so much.

10. Jika terdapat kata ‘either…or’ dan ‘neither…nor’ dalam kalimat, verb atau to be dari
kalimat tersebut tergantung pada kata sesudah kata ‘or’ dan ‘nor’ dalam kalimat
Contoh:
Neither Evi nor Nova goes shopping. Neither Evi nor her children go shopping.

tersebut.

11. Ungkapan jarak (distance), waktu (time), berat, (weight), dan uang (money) predikatnya
dianggap singular karena merupakan satuan bukan jumlah.
Contoh:
Ten dollars is too expensive for the small toy.

12. Judul buku, novel, film, dan permainan dianggap singular.


Contoh:
Titanic is one of the best movies that I ever seen.

13. Kata penghubung and menghubungkan klausa yang setara dan memiliki
kedudukan yang sama.
Contoh:
Both you and him are my bestfriends.

CONTOH SOAL

1. About ten percent of the Earth’s land area area or nearly 5,8 million squares miles … by
glacial ice. (UMPTN/2001/R-C/342)
A. are covered
B. has covered
C. to be covered

3
D. had covered
E. is covered
Pembahasan :
Jika terdapat kata-kata yang menyatakan satuan tertentu dalam sebuah kalimat, predikat
dari kalimat tersebut akan menjadi singular.
Jawaban: E

2. A number of students … complaining about the test. (UM-UGM/2004/121)


A. is
B. are
C. be
D. being
E. been
Pembahasan :
Pilihan A merupakan indefinite article, sehingga predikat yang
mengikuti kata-kata tersebut akan dianggap plural (jamak).
Jawaban: B

3. All doctors in our hospital have their own practice room and each of them … nurse to
assist them. (SPMB/2005/R-II/640)
A. needs
B. they need
C. to need
D. needed
E. need
Pembahasan :
Sekalipun di awal kalimat terdapat kata all doctors, perlu diperhatikan
penggunaan kata each yang harus diikuti oleh plural, namun tetap
dianggap singular. Jadi, pilihan A menjadi jawaban yang paling tepat.
Jawaban: A

4. The shipment of logs … because the bad weather. (UMPTN/1990/R-B/53)


A. they have been posponed
B. has been postponed
C. to be postponed
D. been postponed
E. being postponed

4
Pembahasan :
Frasa the shipment of logs dapat kita ganti dengan menggunakan kata
ganti It. Jadi, predikat yang tepat untuk mengikuti kata it ialah pilihan B.
Jawaban: B

5. None of the beauty contestants ... to wear bikini provided by the commitee.
(SPMB/2004/ R-II/241)
A. refuses
B. are refusing
C. to refuse
D. refuse
E. they refuse
Pembahasan :
Kata None of ... pada kalimat tersebut mengatakan tidak ada satupun
dari para kontestan yang menolak memakai bikini. Kata tersebut
dihitung sebagai singular, seperti kata somebody, anybody, dan lain-lain.
Jawaban: A

5
BAHASA
InggRIS
Chapter 14
Elliptical Construction

Kalimat eliptik adalah kalimat pemendekan dari dua buah kalimat atau lebih dengan cara
menghilangkan beberapa bagian kalimat yang memiliki kesamaan arti asalnya. Kalimat eliptik
digunakan untuk menghindari adanya pengulangan kata yang sama dalam suatu
kalimat. Adapun bagian yang dihilangkan diganti dengan menggunakan bentuk auxiliary
(kata kerja bantu) atau menggunakan kata-kata, seperti so, too, either, neither, dan so on.

A. GABUNGAN SETARA
Merupakan gabungan dua kalimat yang sama-sama positif maupun negatif. Gabungan
setara selalu menggunakan kata penghubung and.
a. Setara Positif (so dan too)
Contoh:
John is smart. The children are smart.
 John is smart and the children are too.
 John is smart and so are the children.
We studied hard. Emily studied hard.
 We studied hard and Emily did too.
 We studied hard and so did Emily.
b. Setara Negatif (neither dan either)
Contoh:
They don’t like sushi. He doesn’t like sushi.

1
 They don’t like sushi and neither does he.
 They don’t like sushi and he doesn’t either.
Tom wasn’t happy. His parents were not happy.
 Tom wasn’t happy and neither were his parents.
 Tom wasn’t happy and his parents were not either.

B. GABUNGAN SETARA BERPASANGAN


Merupakan gabungan dua kalimat yang sama-sama positif dan negatif dengan
menggunakan both… and…, not only… but also… untuk kalimat positif, sedangkan
either… or… dan neither… nor… untuk kalimat negatif.
• Dania loves dancing. Riana loves dancing.
 Both Dania and Riana love
dancing. Jimmy is smart. Jimmy is
humble.
 Jimmy is not only smart but also humble.
• The bus drivers start working today. The taxi drivers start working today.
 Either the bus drivers or the taxi drivers start working today.
• Rini forgot where she put the books.
 It is either in the living room or in her bedroom.
• He isn’t a teacher. He isn’t a doctor.
 He is neither a teacher nor a doctor.
• He doesn’t say rude things. They don’t say rude things
 Neither he nor they say rude things.

C. GABUNGAN SETARA BERLAWANAN


Merupakan gabungan dua kalimat yang berlawanan makna. Umumnya, menggunakan
kata penghubung but, whereas, dan while.
Contoh:
• John likes soccer. Jonathan doesn’t like soccer.
 John likes soccer but (while) Jonathan doesn’t.
• She has a plan to go camping. He doesn’t have a plan to go camping.
 She has a plan to go camping whereas e doesn’t.

D. SUSUNAN ELIPTIK DALAM KLAUSA


Susunan eliptik, biasanya, digunakan dalam dependent dan independent clause bila kedua
subjeknya sama.
a. Mary was poor but she was honest (Mary was poor but honest)
b. However he is busy, he always comes on time to the office. (However busy, he
always comes on time)

2
c. Although she was given everything, she was never satisfied. (Although given
everything, she was never satisfied)
d. I will write a letter. I will write a short story. (I will write a letter and a short story).
e. You can go by bus. You can go by taxi. (You can go by bus or taxi).

CONTOH SOAL

1. “Andy agreed that everybody had to pay Rp50.000,00 for the study tour to Bali.
“ … “ (UMPTN/92/R-B/34)
A. Neither was I.
B. But I’m not.
C. So am I.
D. While I didn’t.
E. I didn’t either.
PEMBAHASAN:
Kalimat tersebut merupakan kalimat eliptik pertentangan yang awal kalimatnya termasuk
kalimat positif dan kalimat selanjutnya kalimat negatif. Jadi, pilihan D menjadi jawaban
yang paling tepat.
Jawaban: D

2. Because of the rain, the teacher couldn’t come on time and … (UMPTN/1991/R-A/63)
A. I neither could.
B. Neither could.
C. Also I could not I.
D. I couldn’t either.
E. So I didn’t.
PEMBAHASAN:
Kalimat eliptik tersebut merupakan kalimat eliptik bermakna negatif yang menggunakan
Modal Past (couldn’t). Jadi, pilihan D menjadi jawaban yang paling tepat.
Jawaban: D

3. Yanti didn’t like the movie and Tati didn’t


either. Means: … (UMPTN/1991/R-C/36)
A. Yanti and Tati didn’t like movie.
B. Yanti didn’t like the movie as much as Tati did.
C. Only Yanti didn’t like the movie.

3
D. Yanti didn’t like the movie but Tati did.
E. One of the two girls didn’t like the movie.
PEMBAHASAN:
Kalimat tersebut merupakan kalimat eliptik yang keduanya sama-sama menyatakan
pernyataan negatif dalam bentuk Past Tense. Jadi, pilihan A menjadi jawaban yang paling
tepat.
Jawaban: A

4. Advertisements both influence people’s spending habits …. (SPMB 2002)


A. their standard of living is also raised
B. but also the raising of their standard of living
C. they can raise their standard of living too
D. and raise their standard of living
E. as well as their raise in their standard of living
PEMBAHASAN:
Kata sambung dalam kalimat eliptik tersebut ialah both. Kata ini yang selalu berpasangan
dengan and. Jadi, pilihan D menjadi jawaban yang paling tepat.
Jawaban: D

5. Tari was punished by the teacher not only because she forgot to bring her book ….
(SPMB 2007)
A. and she didn’t do her homework also
B. but she didn’t do her homework either
C. but also for not doing her homework
D. as well as she didn’t do her homework
E. but also because she didn’t do her homework
PEMBAHASAN:
Kata sambung dalam kalimat eliptik tersebut ialah not only. Kata sambung ini selalu
berpasangan dengan but also. Not only pada soal diikuti oleh adverbial clause yang
menerangkan alasan maka but also juga harus diikuti oleh adverb of reason. Jadi, pilihan
C menjadi jawaban yang paling tepat karena disertai adverb of reason.
Jawaban: C

6. The camping ground where we camped last weekend neither had a facility for
rock climbing practice …. (SPMB 2005)
A. the water was also dirty
B. nor did it have clean water
C. there was no clean water

4
D. we could also not find clean water
E. and the water was dirty too
PEMBAHASAN:
Kata sambung pada kalimat eliptik tersebut ialah neither. Kata sambung ini yang
selalu berpasangan dengan kata nor. Jadi, pilihan B menjadi jawaban yang paling tepat.
Jawaban: B

7. “I don’t like his way of treating patients.”


“… his attitudes towards his subordinates.” (SPMB 2003)
A. You don’t like
B. Whether don’t like
C. Don’t you like
D. I don’t like it either
E. Nor do I like
PEMBAHASAN:
Kalimat tersebut merupakan kalimat eliptik yang bermakna negatif. Kalimat kedua
merupakan suatu persetujuan terhadap kalimat pertama. Dengan kata lain, subjek
(kalimat kedua) merasakan hal yang sama sekaligus memberikan contoh lain untuk
menyetujui pernyataan subjek pertama. Jadi, pilihan E menjadi jawaban yang paling
tepat.
Jawaban: E

8. They have worked hard to prevent excessive government spending, protect


environment, and …. (SPMB 2002)
A. provide good education
B. the provision of quality education
C. providing good education
D. quality education is provided
E. for providing quality education
PEMBAHASAN:
Kalimat tersebut merupakan kalimat eliptik setara yang menggunakan kata
penghubung and. Karena penggunaan and dalam kalimat tersebut diikuti Verb 1,
pilihan A menjadi jawaban yang paling tepat.
Jawaban: A

9. “I heard that the thieves managed to take your electronics”


“Well, not only electronics ….” (SPMB 2004)
A. as well as my precious jewelries
B. but they took my precious jewelries too
C. and also my precious jewelries

5
D. but also my precious jewelries
E. and they took my precious jewelries
PEMBAHASAN:
Dalam kalimat kedua, kalimat eliptiknya menggunakan kata penghubung not only.
Pasangan not only yang paling tepat ialah but also. Jadi, pilihan D menjadi jawaban yang
paling tepat.
Jawaban: D

10. Neither the hotel facilities … stay for guests. (SPMB 2003)
A. and also its service
B. nor its service
C. and not its service
D. or its hotel service
E. both its service
PEMBAHASAN:
Kalimat tersebut merupakan kalimat eliptik yang setara berpasangan neither dan nor.
Jadi, pilihan B menjadi jawaban yang paling tepat.
Jawaban: B

6
BAHASA
InggRIS
Chapter 15 Relative Pronoun

Relative Pronoun seringkali disebut sebagai adjective clause. Relative Pronoun memiliki fungsi
yang sama seperti kata sifat (adjective) dalam sebuah kalimat majemuk (complex sentence),
yaitu menerangkan subjek atau objek yang berupa kata benda. Penggunaan adjective clause
ditandai dengan penggunaan relative pronoun atau kata ganti penghubung yang menunjuk
pada orang atau benda yang dibicarakan dalam kalimat, seperti who, whom, whose, which,
that, in which, dan on which.

A. WHO
Digunakan untuk menjelaskan orang (person) dengan makna aktif. Pola yang digunakan,
yaitu:
Who + P (verb/aux)
Contoh 1:
The man is Catur. He is standing beside the red car.
The man who is standing beside the red car is Catur.

Contoh 2:
He is the singer who sings our favorite songs.

1
B. WHOM
Digunakan untuk menjelaskan orang (person) yang bermakna pasif. Pola yang digunakan,
yaitu:
Whom + S + P (without object)
Contoh 1:
Mira is the girl. Andi hates her.
Mira is the girl whom Andi hates.

Contoh 2:
They are the artist whom the girl adores.

C. WHOSE
Digunakan untuk menggantikan kata ganti milik, seperti my, your, their, our, their, her, his,
dan its. Pola yang digunakan, yaitu:
(Whose + noun) + P
Contoh 1:
The old man looks very sad. His wife is in the hospital.
The old man whose wife is in the hospital looks very sad.

Contoh 2:
Mia whose car is very expensive is an arrogant girl.

D. WHICH
Digunakan untuk menjelaskan benda bukan orang, baik yang yang bermakna aktif
maupun pasif. Pola yang digunakan, yaitu:
Which + P (verb/aux)
Contoh 1:
Miaw is my cat. I feed it every day.
Miaw which I feed every day is my cat.

Contoh 2:
The book which I think is lost is on Ria’s desk.

E. WHERE (In WHICH)


Merupakan penjelas untuk keterangan tempat.
Contoh:
The apartment is so comfortable. We stayed there last year.
The apartment where/in which we stayed last year is so comfortable.

2
F. WHEn (On WHICH)
Merupakan penjelas untuk keterangan waktu.
Contoh:
The month was January. The wedding ceremony was held on it.
The month when/on which the wedding ceremony was held was January.

note:
1. Who menyiratkan frase orang yang diterangkan tersebut berkedudukan sebagai pelaku
(subject) dalam klausa penjelasnya.
2. Whom menyiratkan frasa orang yang diterangkan tersebut berkedudukan sebagai
objek dalam klausa penjelasnya, kecuali jika mendapat preposisi ‘of’ (of whom)
dapat berkedudukan sebagai subjek maupun objek.
3. Whose menyiratkan frasa orang yang diterangkannya dengan frasa benda yang
menjelaskannya berupa hubungan kepemilikan. Whose juga dapat merangkai frasa
benda (frasa bukan orang) selama menunjukkan hubungan kepemilikan, namun
lazimnya kepemilikan selain penunjuk orang dipergunakan of which.
4. That dapat digunakan saat frasa benda yang diterangkannya menunjuk pada
orang/selain orang berkedudukan sebagai subjek maupun objek.

CONTOH SOAL

1. Tom’s father … health hasn’t been so good lately, is in the hospital now. (UMPTn 1990)
A. who is
B. whose
C. his
D. of whom
E. of which
PEMBAHASAn:
Kalimat tersebut memiliki subjek orang (person) yang kondisi kesehatannya
(kesehatan miliknya) tidak terlalu baik. Karena kalimat tersebut menyatakan suatu
kepemilikan, relative pronoun yang paling tepat ialah pilihan B (whose) yang
menyatakan hubungan possessive (kepemilikan).
Jawaban: B

2. This is the room … Churchill was born. (UM-UGM/2004/121)


A. that
B. which
C. in which

3
D. whose
E. of which
PEMBAHASAn:
Pertanyaan pada kalimat tersebut mengarah pada kata the room yang merupakan suatu
tempat. Dalam relative pronoun, kata yang paling tepat untuk menunjukan keterangan
tempat adalah in which/where. Jadi, pilihan C menjadi jawaban yang paling tepat.
Jawaban: C

3. These tourists, … are Japanese, were among the crowd participating in the Dangdut
dance. (SPMB/2006/R-I)
A. there are many
B. many of them
C. whose many
D. many of whom
E. they who
PEMBAHASAn:
These tourists merupakan subject plural (jamak). Kalimat selanjutnya dikatakan banyak
dari mereka  mereka merupakan relative pronoun pengganti objek. Jadi, pilihan D
menjadi jawaban yang paling tepat.
Jawaban: D

4. The doctor is treating a patient … leg was broken in an accident. (UMPTn/1993/R-B)


A. who
B. whose
C. which
D. that
E. whom
PEMBAHASAn:
Kalimat tersebut menyatakan bahwa seorang dokter sedang merawat seorang pasien
yang … (kakinya) yang patah dalam kecelakaan. Karena kalimat tersebut menunjukkan
kepemilikan, jawaban yang paling tepat ialah pilihan B.
Jawaban: B

5. Students … will be awarded scholarships. (UMPTn/199/R-B)


A. who pass with excellent grades
B. they pass with excellent grades
C. they who pass with excellent grades
D. pass with excellent grades
E. to pass excellent grades

4
PEMBAHASAn:
Kalimat tersebut menyatakan bahwa para siswa … yang lulus dengan nilai yang terbaik
akan dianugerahi berbagai beasiswa.Dalam kalimat tersebut, hanya mengacu pada
satu subjek, yaitu students. Jadi, pilihan A menjadi jawaban yang paling tepat.
Jawaban: A

6. The sanitation in this area, … is neglected by the inhabitants, has caused the spread
of many infectious diseases. (SPMB/2005/R-I)
A. the maintenance of which
B. that maintenance
C. of which the maintenance
D. which maintenance
E. the maintenance of it
PEMBAHASAn:
The sanitation in this area … menunjukkan pada suatu sistem dan bukan kata ganti orang.
Jadi, relative pronoun yang paling tepat ialah pilihan C.
Jawaban: C

7. “I heard that Lola’s marriage has been arranged by her family.”


“Really? So, she is marrying a man … she hardly knows.” (SPMB 2007)
A. whom
B. to whom
C. whose
D. that is
E. which
PEMBAHASAn:
Relative pronoun pada adjective clause kalimat tersebut yang paling tepat ialah whom. Ini
karena terdapat dua orang dalam kalimat tersebut, yakni she sebagai subjek dan a man
sebagai objek.
Jawaban: A

8. The singing contest participants, only five of … will be selected to enter the grand final
are now waiting for the Jury’s decision. (SPMB 2005)
A. whom
B. whose
C. them
D. which
E. these

5
PEMBAHASAn:
Relative pronoun yang diikuti oleh predikat berfungsi untuk menjelaskan orang (person),
yaitu who. Namun, kalimat tersebut memiliki sebuah preposisi of. Jadi, who tidak bisa
kita gunakan, tetapi whom yang digunakan (pilihan A).
Jawaban: A

9. “You look worried. What’s the


matter?” “I don’t know ….” (UMPTn
2001)
A. where did I leave my car keys
B. when my car keys are left
C. where I left my car keys
D. when did I leave my car keys
E. where are my car keys
PEMBAHASAn:
Pada kalimat pertama, dapat dilihat bahwa kalimat tersebut menunjukkan sebuah klausa
yang diminta untuk menunjukkan tempat. Ini karena tempat meletakkan kunci
mobilnya yang membuat orang khawatir, bukan waktu meletakkan kunci tersebut.
Jadi, kata penghubung yang paling tepat ialah pilihan C.
Jawaban: C
10. Participants of the state university entrance test, … are high school graduates from Java
are completing for a limited number of seats. (SPMB 2002)
A. which most of them
B. a greater number
C. most of whom
D. the majority
E. a lot of them
PEMBAHASAn:
Kalimat tersebut menggunakan relative pronoun yang diikuti oleh auxiliary (are) dan
berfungsi untuk menjelaskan orang (participants of the state university entrance test)
ialah who. Namun, kalimat tersebut memiliki sebuah preposisi (kata depan) sebelum
relative pronoun. Relative pronoun yang lebih tepat digunakan ialah whom. Jadi, pilihan C
menjadi jawaban yang paling tepat.
Jawaban: C

6
BAHASA
InggRIS
Chapter 16 Noun Clause

Noun clause adalah clause (anak kalimat) memiliki fungsi yang sama seperti kata benda, di
antaranya subjek, objek, maupun pelengkap. Noun clause biasa diawali dengan kata tanya
(when, where, why, how, who, whom, what, which, whose), whether, if, dan that. Noun clause
dapat berfungsi menjadi tiga hal, yaitu subjek, objek, dan pelengkap.

A. NOUN CLAUSE SEBAGAI SUBYEK


Contoh:
It is strange. What I dreamed last night is strange.

Dari kalimat pertama di atas, it berfungsi sebagai subjek. Kita dapat mengganti it dengan
klausa what I dreamed last night sebagai noun clause yang berfungsi sebagai subjek.
Contoh:
Where she lives is still unclear.
That they passed the exam surprise us.

B. NOUN CLAUSE SEBAGAI OBYEK


Biasanya, muncul setelah kata kerja: hope, agree, ask, decide, promise, remember, forget,
understand, believe, explain, imagine, know, learn, say, see, think, dan lain-lain.
Contoh:
Mr. Chairil agrees that he will go to Yogyakarta tomorrow.

1
The boy will give what his girlfriend asks.
The students have to do what their teacher assigned them.

Perlu diingat bahwa kalimat dalam noun clause haruslah berupa affirmative (pernyataan).
Contoh:
Where do you live?  I know where you live.

C. NOUN CLAUSE SEBAGAI PELENGKAP (COMPLEMENT)


Contoh:
This tour is what I have been dreaming.
This is where I work every day.

CONTOH SOAL

1. … is a big surprise to his colleagues. (SPMB/2002/R-II)


A. Tanto has been promoted to replace the retired Financial Manager
B. Do you know Tanto has been promoted to replace the retired Financial Manager
C. I hear Tanto has been promoted to replace the retired Financial Manager
D. That Tanto has been promoted to replace the retired Financial Manager
E. Why was Tanto promoted to replace the retired Financial Manager
PEMBAHASAN:
Kalimat tersebut merupakan noun clause sebagai subjek. Jadi, pilihan D menjadi jawaban
yang paling tepat.
Jawaban: D

2. “What is the most worrying effect of the economic crisis?”


“… children are deprived of nutritious foods necessary for their physical and intellectual
development.” (UMPTN 2000)
A. why
B. that
C. what
D. which
E. when
PEMBAHASAN:
Dalam kalimat tersebut, kata what menanyakan benda (noun). Respons yang paling tepat
harus berupa noun clause. Dan kata penghubung yang paling tepat unuk noun clause
adalah (B) that.
Jawaban: B

2
3. “What did the speaker say at the seminar?”
“… while they are watching TV is important.” (SPMB 2007)
A. That you should accompany your children
B. You should accompany your children
C. That accompanying your children
D. Accompany your children
E. You are accompanying your children
PEMBAHASAN:
Pada kalimat tersebut, kata what digunakan untuk menanyakan perihal/benda (noun).
Jadi, piihan A menjadi jawaban yang tepat karena pola noun clause sebagai subjek
(question word + S + P).
Jawaban: A

4. When Tomoko a Japanese student was asked … of the museum, she quickly replied
that it had taught her a great deal about the history of Kraton Yogya. (SPMB 2002)
A. what did she think
B. about thinking
C. what she thought
D. if she was thinking
E. what she did think
PEMBAHASAN:
Kalimat tersebut merupakan noun clause. Unsur bahasa yang digunakan sebelum
preposisi of harus berupa noun atau noun clause. Susunan yang paling tepat dengan
pola noun clause, yaitu (question word + S + P). Jadi, pilihan C menjadi jawaban yang
paling tepat.
Jawaban : C

5. … is not yet known yet. (SPMB 2002)


A. Can he be involved in the trade of narcotics
B. He is involved in the trade of narcotics
C. Whether he is involved in the trade of narcotics
D. When he is involved in the trade of narcotics
E. Why he is involved in the trade of narcotics
PEMBAHASAN:
Kalimat tersebut membutuhkan noun clause sebagai subjek kalimat. Namun, berdasarkan
kalimat tersebut, subjek belum dapat dipastikan bahwa dia terlibat dalam
perdagangan narkotika. Jadi, pilihan A (whether = apakah) menjadi jawaban yang paling
tepat.
Jawaban: C

3
6. “Have you seen Mr. Imam about our proposal?”
“Yes, I have and I asked him …” (SPMB/2006/R-I/411)
A. Was our proposed budget feasible.
B. That our proposed budget feasible..
C. Whether our proposed budget was feasible.
D. How feasible was our budget.
E. Our proposed budget was feasible.
PEMBAHASAN:
Kalimat tersebut berpola indirect object yang berupa noun clause. Jadi, pilihan C menjadi
jawaban yang tepat.
Jawaban: C

7. “Has the mother finally decided what to buy for herself?”


“I don’t know. Let’s ask her about what ….” (SPMB/2006/R-II/310)
A. does she want to buy
B. to buy
C. is buying
D. does she buy
E. she wants to buy
PEMBAHASAN:
Sesuai dengan pola noun clause yang ada, pola kalimat tersebut akan menjadi (question
word + S + P). Jadi, pilihan E menjadi jawaban yang paling tepat.
Jawaban: E

8. Through a wide variety of technological innovations that include farming methods and
sanitations, as well as the control of these deadly disease, we found the way to reduce
the rate … we die, creating a population explosion. (SIMAK UI 2013)
A. that
B. why
C. how
D. when
E. at which
PEMBAHASAN:
Kalimat tersebut merupakan noun clause yang mengacu pada kata the way, yakni cara
(noun) untuk mengurangi rating/tingkat cara kita meninggal atau tingkat kematian. Jadi,
pilihan C menjadi jawaban yang paling tepat.
Jawaban:C

4
9. Dr. Alison lane, the director of the Cairns Marine Science Institute, contends … we
need marine parks for scientific research. (SIMAK UI 2013)
A. how
B. why
C. when
D. where
E. that
PEMBAHASAN:
Kalimat tersebut merupakan kalimat pernyataan yang berupa noun clause. Untuk, mengisi
bagian rumpang … kita membutuhkan taman laut di bawah air untuk keperluan penelitian
maka pilihan E menjadi jawaban yang paling tepat.
Jawaban: E

10. … Bali is a beautiful island, rich in culture, and has been confirmed by tourists from
around the world. (UMPTN/1999/R-A)
A. Bali
B. Whether Bali
C. If Bali
D. That Bali
E. How Bali
PEMBAHASAN:
Kalimat tersebut merupakan noun clause yang berfungsi sebagai subjek. Jadi, pilihan D
menjadi jawaban yang paling tepat.
Jawaban: D

5
6
BAHASA
InggRIS
Chapter 17 Participle

Participle adalah kata kerja (verb) yang berfungsi sebagai kata sifat (adjective). Participle
berfungsi untuk menerangkan sifat suatu kata benda (noun).

Dalam bahasa Inggris, dikenal dua macam participle, yakni present participle (aktif) dan past
participle (pasif). Keduanya memiliki arti serta bentuk yang berbeda. Kedua bentuk participle
tersebut dapat digunakan untuk menerangkan kejadian yang berlangsung pada present,
past, atau bahkan future.

A. PRESENT PARTICIPLE
Kata kerja dalam bentuk –ing dan artinya menunjukkan pada aktifitas yang bersifat
aktif (melakukan pekerjaan) ataupun sedang dilakukan. Contohnya, doing, eating, dan
sweeping. Penggunaan present participle (active) untuk:
a. Membentuk Continuous Tense
Contoh:
We are studying English now.
b. Setelah Verb of Sense: Catch, Find, and SLOW N FLASH (See, Listen, Observe, Watch,
Notice, Feel, Look At, Smell, Hear)
Contoh:
We saw Nurhayati cooking in our kitchen.

1
c. Noun Modifier/Adjective
Contoh:
The walking stick is really useful for that old man.
d. Clause Modifier
1. Dua peristiwa yang terjadi secara bersamaan yang dilakukan oleh pelaku
yang sama.
Contoh:
Sleeping, he felt down from his chair in the class.
While he was sleeping, he felt down from his chair in the class.
2. Dua peristiwa yang terjadi secara berurutan yang dilakukan oleh pelaku
yang sama dan bermakna telah atau after.
Contoh:
Having finished the homework, they went home.
After they had finished the homework, they went home.
3. Peristiwa yang merupakan penyebab terjadinya peristiwa peristiwa lain
pada pelaku yang sama, yakni dengan menambahkan kata being yang
berarti karena.
Contoh:
Being old, my grandmother always stays at home all the time.
Karena sudah tua, nenek saya tinggal di rumah sepanjang waktu.

B. PAST PARTICIPLE
Kata kerja bentuk ke-3 (Verb 3) yang berarti telah dan menunjuk pada aktivitas yang
bersifat pasif (dikenai pekerjaan). Contohnya, broken, closed, dan read. Penggunaan Past
Participle (Passive) untuk:
a. Membentuk Kalimat Pasif
Contoh:
Jakarta, surrounded by tall buildings, is very crowded nowadays.
b. Noun Modifier
Contoh:
The broken window has been fixed.
c. Clause Modifier
1. Sebab-akibat pada pelaku yang
sama. Contoh:
Accused of dishonesty by the media, the minister decided to resign.
 Because he is accused of dishonesty by the media, the minister decided
to resign.

2
2. Peristiwa bersamaan yang terjadi pada pelaku yang sama.
Contoh:
Stolen some jewelries, the thief was arrested by the policeman.
 When the thief was stolen some jewelries, he was arrested by the policeman.

CONTOH SOAL

1. … a few million rupiahs, he went on a tour to Europe. (UMPTN/1990/ R-A/53)


A. Saved
B. Have saved
C. He has saved
D. Having saved
E. After he saves
PEMBAHASAN:
Kalimat tersebut bermakna, … beberapa juta rupiah, dia pergi mengikuti tour ke Eropa.
Kalimat tersebut termasuk kalimat present participle karena bermakna setelah menabung.
Jadi, pilihan D menjadi jawaban yang paling tepat.
Jawaban: D

2. … by not more than 40 families, Kampung Naga is a unique village in West Java.
(UMPTN/1993/R-C/23)
A. Inhabit
B. To inhabit
C. Inhabited
D. Inhabitant
E. Inhabiting
PEMBAHASAN:
Salah satu ciri kalimat pasif, yaitu penggunaan kata by. Sama seperti kalimat yang
menggunakan Past Participle, pilihan C menjadi jawaban yang paling tepat.
Jawaban: C

3. Having read the document carefully, … (UM UGM/2003/121)


A. The manager signed it.
B. The manager’s signature was affixed.
C. The document is signed by the manager.
D. It was signed by the manager.
E. The document that is signed by the manager.

3
PEMBAHASAN:
Yang dibutuhkan untuk melengkapi kalimat tersebut, yaitu subjek. Jadi, jawaban yang
paling tepat untuk melengkapi Present Participle tersebut ialah pilihan A.
Jawaban: A

4. Ornamented with beautiful traditional Balinese carving, … (UMPTN/2001/R-C/341)


A. Many foreign tourists are curious to visit the house.
B. The antiquity of the house is admired by many tourists.
C. Tourists guides often bring their guests to visit the house.
D. The owner of the house allows tourist to visit and admire its beauty.
E. The house of a local resident becomes an object of tourism.
PEMBAHASAN:
Kalimat tersebut memerlukan subjek untuk melengkapi kalimatnya. Bagian awal
dalam kalimat tersebut berarti ornamen yang dihiasi oleh ukiran tradisional Bali. Untuk
lengkapi kalimat tersebut, subjek harus diisi dengan sesuatu yang mengacu pada
sebuah benda yang bisa dihiasi ornamen bali. Jadi, pilihan E menjadi jawaban yang paling
tepat.
Jawaban: E

5. Holding posters with various messages for the Board , the factory workers are claiming
better working condition. The underlined words mean … posters with various messages.
(SPMB/2003/R-II/110)
A. While they are holding
B. Because they held
C. After they had held
D. Since they have held
E. Before they hold
PEMBAHASAN:
Kalimat tersebut merujuk pada suatu kejadian ketika para pekerja pabrik (buruh)
memegang poster yang berisi berbagai macam pesan yang ingin mereka
sampaikan kepada dewan, sambil menuntut lingkungan kerja yang lebih baik lagi.
Karena kejadian tersebut berlangsung bersamaan, pilihan A menjadi jawaban yang paling
tepat.
Jawaban: A

6. Trying hard to win the beauty contest, Ita neglected all her school assignments.
(UMPTN/1993/R-A/33)
A. If she tried
B. Although she tried
C. Before she tried
D. Since she tried

4
E. After she tried

5
PEMBAHASAN:
Kalimat tersebut merupakan kalimat present participle yang berarti karena berusaha keras
untuk menang dalam sebuah kontes kecantikan, Ita mengabaikan semua tugas sekolahnya.
Semua pilihan jawaban merupakan kata penghubung, namun yang paling tepat
dan menyatakan penyebab ialah pilihan D (since yang berarti karena)
Jawaban: D

7. Two of my friends … in the school fight last weekend have been arrested by the police.
(UMPTN/1993/R-C/23)
A. they were involved
B. involved
C. to be involved
D. involving
E. were involved
PEMBAHASAN:
Kalimat tersebut termasuk kalimat past participle yang ditandai kata last weekend
(kejadiannya minggu lalu). Kalimat tersebut juga bermaknanya pasif. Jadi, pilihan B
menjadi jawaban yang paling tepat dengan menggunakan verb 3 (involved).
Jawaban: B

8. Having finished their exam, … (UMPTN/2000/R-A/46)


A. The books must returned to the library.
B. The library requires the books to be returned.
C. The library requires students to return the books.
D. The students must return the books to the library.
E. It is required that students return the books to the library.
PEMBAHASAN:
Kalimat tersebut memerlukan subjek untuk melengkapi kalimatnya. Dalam kalimat
tersebut, their merujuk pada sekumpulan orang yang telah selesai ujian. Sekumpulan
orang ini dapat diartikan sebagai pelajar. Jadi, pilihan D menjadi jawaban yang paling
tepat.
Jawaban: D

9. In Biology, a cell is defined as the smallest unit of life … all the components required for
independent existence. (UM-UGM/2003/121)
A. contains
B. is contained
C. it contains
D. containing
E. to be contained

6
PEMBAHASAN:
Kalimat tersebut merupakan kalimat dengan pola present participle yang bermakna aktif
(me-), Dalam ilmu Biologi, sel dapat didefinisikan sebagai unit terkecil dalam kehidupan
… (yang mengandung) semua komponen kehidupan yang dibutuhkan. Jadi, pilihan D
menjadi jawaban yang paling tepat.
Jawaban: D
10. Inspired by the recent natural disasters, … (SPMB/2005/R-II/270)
A. Sad and sentimental songs are composed.
B. Composers produced sad and sentimental songs.
C. There are compositions of sad and sentimental songs.
D. The composition includes sad and sentimental songs.
E. The result is the composition of sad and sentimental songs.
PEMBAHASAN:
Kalimat tersebut termasuk kalimat present participle yang membutuhkan subjek. Subjek
ini ialah orang (pelaku) yang menciptakan lagu sedih dan sentimental. Jadi, pilihan B
menjadi jawaban yang tepat.
Jawaban: B

7
BAHASA
InggRIS
Chapter 18 Direct-Indirect Speech

Direct Speech adalah suatu ucapan/pernyataan langsung dari pembicara dengan cara
mengulangi kalimat penutur apa adanya tanpa adanya perubahan. Penulisan kutipan
langsung dalam direct speech selalu diapit oleh dua tanda kutip “...” yang diawali oleh tanda
koma. Indirect Speech (Reported Speech) atau kalimat tidak langsung adalah kalimat yang
tidak langsung diucapkan oleh pembicaranya, melainkan diceritakan kembali oleh orang lain.
Direct-Indirect Speech terdiri dari tiga macam, yaitu kalimat pernyataan, perintah, dan tanya.

A. KALIMAT PERNYATAAN (STATEMENT)


a. Jika kalimat pengantarnya present tense maka tenses dalam indirect speech
tidak berubah, yang berubah hanya pronoun dan adverbial-nya saja.
Contoh:
Direct  He says, “I love you.”
Indirect  He says that he loves me.
b. Jika kalimat pengantarnya past tense maka tenses dalam indirect speech berubah,
termasuk pronoun dan adverbial-nya.
Contoh:
Direct  She says, “I cook every day.”
Indirect  She said that she cooked every day.

1
B. KALIMAT PERINTAH (COMMAND)
a. Positive Command (+ to
V1) Contoh:
Direct  Toni said, “Open the door now!”
Indirect  Toni asked me to open the window then.
b. Negative Command (not + to
V1) Contoh:
Direct  Sarah says, “Don’t love me anymore!”
Indirect  Sarah ordered me not to love her anymore.

C. KALIMAT TANYA (INTERROGATIVE)


a. Dimulai dengan Yes/No Questions maupun auxiliary verbs, seperti do, did, does, is,
am, are, was, were, have, has, had, will, can, may, shall must, dan have to. Kita harus
menggunakan whether/if dan mengubah kalimat tanya menjadi kalimat berita
(pernyataan).
Contoh:
Direct  Ahmad asked me, “Do you like an ice cream?”
Indirect  Ahmad asked me whether/if I liked an ice cream.
b. Pertanyaan yang dimulai dengan Question Words (5W1H), whom, dan which. Kita
harus mengubah kalimat tanya menjadi kalimat berita.
Contoh:
Direct  Fifi asked me, “Who is your teacher?”
Indirect  Fifi wanted to know who was my teacher.

Notes:
1. Ciri kalimat langsung selalu diapit oleh tanda kutip “...”
2. Pattern: Present  Past
Past  Past Perfect
3. Keterangan waktu dan keterangan tempat ikut berubah.

CONTOH SOAL

1. The secretary asked me … with Mr. Sigit. (UMPTN/1990/R-A/53)


A. did I have an appointment
B. how was my appointment
C. whether I had an appointment
D. when is your appointment
E. that I had an appointment

2
PEMBAHASAN:
Kalimat tersebut merupakan kalimat tidak langsung yang menggunakan pola
interrogative (kalimat tanya). Awalnya, kalimat ini termasuk kalimat langsung lalu
diubah menjadi kalimat tidak langsung. Setelah itu, berubah dari kalimat pertanyaan
menjadi kalimat pernyataan. Jadi, pilihan A menjadi jawaban yang tepat
Jawaban: A

2. The foreman said, “Finish your task and do it


well.” Means: … (UMPTN/1993/R-C/23)
A. The foreman asked me to finish his task and to do it well.
B. The foreman ordered me to finished my task and to do it well.
C. The foreman asked me to finish my task and do it well.
D. The foreman told me to finish his task and do it well.
E. The foreman said that I finished my task and do it well.
PEMBAHASAN:
Petikan kalimat langsung tersebut menggunakan kata penutur past tense yang
ditandai The forman said. Kalimat yang diucapkannya itu termasuk kalimat perintah
positif (positive command). Jadi, pilihan B menjadi jawaban yang paling tepat karena
bentuk indirect speech-nya akan menjadi (+ to V1).
Jawaban: B

3. John said to me: “Why do they go abroad so often?”


John asked me … (UMPTN/1999/R-B/12)
A. If they went abroad so often.
B. Why they went abroad so often.
C. Why did they go abroad so often.
D. Why do they go abroad so often.
E. Did they go abroad so often.
PEMBAHASAN:
Kalimat tersebut termasuk direct speech dalam bentuk pertanyaan dengan
menggunakan kata tanya why. Sesuai dengan pola kalimat indirect speech dengan
Question Word (Question Word + S + P), pilihan B menjadi jawaban yang paling tepat.
Jawaban: B

4. “I have been working here for ten years,” Vira


told. Means: … (UMPTN/1991/R-B/31)
A. Vira told that she had worked here for ten years.
B. Vira told that she had been working here for ten years.
C. Vira told that she had been working there for ten years.

3
D. Vira told that she has been working there for ten years.
E. Vira told I had been working there for ten years.
PEMBAHASAN:
Kalimat tersebut merupakan direct speech dalam bentuk present continuous tense
yang ditandai I have been working. Jadi, indirect speech-nya akan berubah dari present
menjadi past dan keterangan tempat menjadi ikut berubah. Oleh karena itu, pilihan C
menjadi jawaban yang paling tepat.
Jawaban: C

5. My friend said to me: “Can I find you a


hotel?” Means: … (UMPTN/1994/R-C/35)
A. My friend asked me if I could help him find a hotel.
B. I wondered if my friend could help me find a hotel.
C. My friend said that I could help him find a hotel.
D. My friend asked me to find a hotel for him.
E. My friend asked me whether he could help me find a hotel
PEMBAHASAN:
Kalimat tersebut merupakan direct speech dalam bentuk pertanyaan dengan
menggunakan modal can. Karena kalimat tersebut merupakan kalimat pertanyaan,
kita harus menggunakan whether/if sebagai penutur untuk indirect speech-nya dan
mengubah kata gantinya. Jadi, pilihan E menjadi jawaban yang paling tepat.
Jawaban: E

6. My father said: “Don’t waste your money on


cigarettes!”. This means … (UMPTN/1997/R-B/57)
A. My father advised me not to waste my money on cigarettes.
B. My father told me that I didn’t waste my money on cigarettes.
C. My father asked me if I had wasted my money on cigarettes.
D. My father allowed me to waste my money on cigarettes.
E. My father said that I didn’t waste my money on cigarettes.
PEMBAHASAN:
Kalimat tersebut merupakan direct speech dalam bentuk imperative yang negatif.
Pola indirect speeech-nya akan menjadi (not + to V1). Jadi, pilihan A menjadi
jawaban yang paling tepat
Jawaban: A.

4
7. Achmad said to me: “Do you close the windows at night?”
The indirect form is: Achmad asked me … at night. (UMPTN/1993/R-B/53)
A. that I closed the windows
B. if I closed the windows
C. whether you close the windows
D. when I closed the windows
E. that you closed the windows
PEMBAHASAN:
Kalimat tersebut merupakan direct speech dalam bentuk Yes/No Question. Untuk pola
indirect speech, kita harus menggunakan kata whether/if serta mengubah pronoun-nya.
Jadi, pilihan B menjadi jawaban yang paling tepat.
Jawaban: B

8. “What did he cashier of the bookstore tell you?”


“… with a credit card.” (UMPTN/1991/R-B/31)
A. Not paying
B. Not pay
C. Let’s not pay
D. Not to pay
E. Did not pay
PEMBAHASAN:
Kalimat tersebut merupakan kalimat pernyataan dalam bentuk imperative yang bermakna
negatif. Jadi, pilihan D menjadi jawaban yang paling tepat.
Jawaban: D

9. “Don’t make noise children!” she said. (UMPTN/1992/R-C/33)


A. She told the children don’t make noise
B. She said the children didn’t make noise
C. She didn’t say the children should make noise
D. She told the children not to make noise
E. She didn’t the children to make noise
PEMBAHASAN:
Kalimat tersebut merupakan kalimat imperative negative yang memiliki pola (not + to
V1). Jadi, pilihan D menjadi jawaban yang paling tepat.
Jawaban: D

5
10. Which of the following sentences has the correct punctuation?
A. The teacher told the students,“Don’t use pencils in exams!”
B. The teacher told the students: “don’t use pencils in exams!”
C. The teacher told the students “Don’t use pencils in exam.”
D. The teacher told the students don’t use pencils in exam.
E. The teacher told the students; don’t use pencils in exams.
PEMBAHASAN:
Punctuation adalah penggunaan tanda baca dan penulisan yang tepat dalam sebuah
kalimat. Jadi, kalimat yang penulisannya paling tepat adalah pilihan A, The teacher told
the students, “Don’t use pencils in exams!”.
Jawaban: A

6
BAHASA
InggRIS
Chapter 19
Gerund and Infinitives

A. GERUND
Gerund adalah kata kerja dalam bentuk –ing yang berfungsi sebagai kata benda. Gerund
bisa digunakan sebagai subjek, objek, complement (pengganti), dan modifier (penjelas)
kata benda. Kata kerja yang biasa diikuti oleh gerund, seperti enjoy, appreciate, avoid,
delay, mind, quit, postpone, finish, forgive, fancy, prevent, keep, consider, like, can’t stand,
suggest, regret, admit, appreciate, continue, deny, escape, excuse, imagine, dan resist. Kata
kerja setelah possesive adjective, seperti my, your, his, her, our, dan their.
Contoh:
They have finished their writing.

Secara umum, gerund terdiri dari tiga macam:


a. Gerund Sebagai Subjek
1. Exercising makes me healthy and fresh.
2. Singing and dancing are good for relaxing.

b. Gerund Sebagai Objek


1. She enjoys writing.
2. Do you mind helping me?

1
c. Gerund Sebagai Objek dari Preposisi
Preposisi adalah kata depan, contoh in, on, at, up, for, after, before, with, without, of,
to, from, dan by with. Preposisi harus diikuti oleh gerund (verb –ing)
1. Thank you for coming.
2. How about going to the cinema?
3. After studying English, we went home.

Penggunaan Gerund lainnya:


a. Membentuk kata benda (noun
phrase) Contoh:
I swim in the swimming pool.
b. Digunakan setelah frasa berikut, yaitu can’t help, can’t stand, dan no use
Contoh:
I can’t stand seeing you with her.
c. Sebagai objek kata kerja: need, want, require, dan worth (makna pasif)
Contoh:
I have read the book. I think it’s worth buying it.
d. Beberapa adjective/verb + preposisi yang harus diikuti oleh gerund, misalnya
approve of, afraid of, interested in, think about, tired of, look forward to, depend on,
insist on, give up, worry about, fond of, object to, get used to, dan get accustomed to.

B. INFINITIVE
Infinitive adalah kata kerja dasar (Verb 1). Infinitive dibagi menjadi dua bagian: infinitive
with to (to infinitive) dan without to (bare infinitive).

a. Penggunaan To Infinitive
1. Sebagai subiek juga
objek Contoh:
To drive fast needs a lot of practice.
They want to study English every day.

2. Sebagai modifier (penjelas)


• Kata sifat untuk menerangkan noun
Contoh:
I know the answer to solve the problem.
• Penjelas Kata Benda
Contoh:
We need a lot of water to drink.

2
• Penjelas kata kerja
Contoh:
The principal was invited to open the ceremony.
• Penjelas kata sifat
Contoh:
Dinda is too young to drive a car.
• Penjelas kata tanya atau frasa dengan kata tanya (5W1H)
Contoh:
I was interested where to go with you.
3. Kata kerja yang diikuti to infinitive, seperti promise, ask, decide, beg, would like,
advise, instruct, hope, permit, allow, invite, warn, force, intend, want, expect,
allow, propose, tell, teach, learn, dan urge.
Contoh:
I want to buy a new car next week.

b. Bare Infinitive
Bare infinitive digunakan untuk:
1. Setelah modal auxiliary, yaitu can, could, may, might, will, would, shall, should, ought
to, dan must.
Contoh:
We must study hard to get the best universities.
2. Setelah verb of perception (kata kerja yang berhubungan dengan panca indera), yaitu
see, hear, feel, smell, make, help, observe, let, dan listen to.
Contoh:
They saw him cry at night.

Note:
Kata kerja yang bisa diikuti V-ing maupun To Infinitive:
a. Tidak ada perbedaan makna: advise, begin, continue, dislike, hate, intend, like, love, prefer,
propose, dan start.
b. Ada perbedaan makna:
S + forget/remember/stop/forget + V-ing  pekerjaan sudah dilakukan
Contoh:
I forget giving her letter. (lupa telah memberi)
S + forget/remember/stop/forget + to inf  pekerjaan belum dilakukan
Contoh:
I forget to give her letter. (lupa belum memberi)

3
CONTOH SOAL
1. My brother, who was not used to … a room with some else, was quite unhappy when
he couldn’t have his own room in the dormitory. (UMPTN/1997/R-A)
A. sharing
B. share
C. be sharing
D. be shared
E. being shared
PEMBAHASAN:
Dalam kalimat tersebut, setelah relative pronoun terdapat to be was yang umumnya diikuti
oleh gerund (v-ing). Jadi, pilihan A menjadi jawaban yang paling tepat.
Jawaban: A

2. “What is that you want us to do?”


“… all the lights when you leave the office.” (SPMB/2004/R-II)
A. Turning off
B. For turning off
C. You turn off
D. To turn off
E. We turn off
PEMBAHASAN:
Yang dibutuhkan dalam kalimat tersebut ialah subjek yang memiliki makna
mematikan semua lampu ketika kamu meninggalkan kantor. Jadi, pilihan A menjadi
jawaban yang paling tepat karena menggunakan gerund sebagai subjek.
Jawaban: A

3. “Why don’t you want Amir to be the team leader:”


“Well, I don’t like … people around!” (SPMB/2004/R-I/440)
A. his ordering
B. why he orders
C. he is ordering
D. he orders
E. with his ordering
PEMBAHASAN:
Kata like merupakan salah satu kata yang umumnya diikuti oleh gerund. Kalimat tersebut
termasuk salah satu kalimat yang menggunakan gerund sebagai objek. Jadi, pilihan A
menjadi jawaban yang paling tepat.
Jawaban: A

4
4. We were astonished at … a good score for his TOEFL in spite of his poor English.
(UMPTN/1992/R-A)
A. he could get
B. is being able to get
C. to a able to get
D. being able to get
E. be able to get
PEMBAHASAN:
Pada kalimat tersebut, terdapat sebuah preposisi at setelah kata sifat. Setelah
penggunaan preposisi, biasanya diikuti oleh gerund. Jadi, pilihan B menjadi jawaban
yang paling tepat.
Jawaban: B

5. I remember … in this town when traffic wasn’t too heavy yet. (UMPTN/1991/R-C)
A. be driving
B. were drive
C. drive
D. our driving
E. to drive
PEMBAHASAN:
Kalimat tersebut menyatakan bahwa subjek mengingat saat dia melintasi jalanan di kota
tersebut, lalu lintas belum terlalu padat. Kalimat tersebut bermakna dia telah melakukan
hal tersebut. Jadi, pilihan D menjadi jawaban yang paling tepat dengan menggunakan
gerund.
Jawaban: D

6. “Where is Budi?”
“Oh. I completely forgot ….” (UMPTN/1992/R-A)
A. to invite
B. not inviting
C. invite
D. not to invite
E. inviting
PEMBAHASAN:
Pada kalimat tersebut, subjek menyiratkan dia lupa belum mengundang Budi ke acara
tersebut. Jadi, pilihan A menjadi jawaban yang paling tepat karena pekerjaan
tersebut belum dilakukan.
Jawaban: A

5
7. My friend Hasan is considering … school before finishing it because the financial reports.
(UMPTN/1997/R-C)
A. he leaves
B. about leaving
C. he wants to leave
D. in leaving
E. leaving
PEMBAHASAN:
Kalimat tersebut menggunakan kata considering yang umumnya diikuti oleh gerund. Jadi,
pilihan E menjadi jawaban yang paling tepat.
Jawaban: E

8. The motion pictures originated when a series of still photograps were spliced and
viewed in rapid succession … the illusion of movement and continuity. (UM-UGM 2004)
A. for creation
B. in the creation of
C. to be created by
D. to create
E. created
PEMBAHASAN:
Kalimat tersebut berarti gambar yang bergerak dimulai ketika foto-foto yang dihasilkan
dan digambarkan … ilusi bergerak dan berkelanjutan. Untuk melengkapi bagian rumpang
itu, kita harus mengisinya dengan kata kerja (V1) infinite with to. Jadi, pilihan D menjadi
jawaban yang paling tepat.
Jawaban: D

9. People are concerned about the depletion of energy sources. However, they should
not forget … another important thing. (SPMB/2002/R-II)
A. to protect
B. it is protecting
C. they protect
D. having to protect
E. protected
PEMBAHASAN:
Kalimat kedua tersebut merupakan kalimat pernyataan yang bermakna negatif karena
berarti namun demikian, mereka seharusnya tidak melupakan hal penting lainnya. Setelah
penggunaan kata not, to + infinivite (A) to protect untuk melengkapinya. Jadi, pilihan A
menjadi jawaban yang tepat.
Jawaban: A

6
10. “While living in Bogor, we always went to school on foot.”
This means that we … on foot while we were living in Bogor. (UMPTN/2001/R-B)
A. go to school
B. are going to school
C. used to go to school
D. are used to going to school
E. liked to go to school
PEMBAHASAN:
Kalimat tesebut menyatakan kebiasaan yang dilakukan di masa lampau dan tidak lagi
dilakukan saat ini. Jadi, pilihan C menjadi jawaban yang paling tepat.
Jawaban: C

7
8
BAHASA
InggRIS
Chapter 20
Derivatives

Derivatives diartikan sebagai kata jadian, yaitu kata yang mengalami perubahan jenis karena
mendapat imbuhan tertentu. Contohnya, sebuah kata benda yang terjadi karena sebuah
kata kerja mendapat imbuhan tertentu. Dengan kata lain, derivatives dapat juga dikatakan
sebagai kata turunan. Dalam bahasa Inggris, awalan (prefix), akhiran (suffix), maupun gabungan
awalan dan akhiran (confix) dapat mengubah makna dan fungsi suatu kata.

Dalam derivatives, terdapat dua macam pola dasar, yakni:


Adjective + Noun
Adverb + Verb

Adjective: Noun:
al  natural ment  development
able  capable ance/ence  maintenance
less  careless ness  happiness
ive  creative ion/tion  action
ior  superior ist  artist
ty  healthy ship  friendship
ful  beautiful hood  childhood
ous  famous or/er  actor/teacher
ish  childish ture  failure
ty  ability

1
Adverb: Verb:
Adjective + (-ly): slowly Verb 1 – Verb 2 – Verb 3 – Verb ing
(-ly) + Verb 3: interested -en: darken
-fy: beautify
en-: enlarge
-ize: apologize

CONTOH SOAL

1. Indonesian batik is … recognized. (UMPTN/1991/R-A/63)


A. international
B. internationalized
C. internationalization
D. internationally
E. internationalizing
PEMBAHASAN:
Bentuk is … recognized merupakan passive verb. Umumnya, verb dipasangkan dengan
adverb untuk melengkapinya. Jadi, pilihan D menjadi jawaban yang paling tepat.
Jawaban: D

2. If you use a word-processor, you can easily … another paragraph to your composition.
(UMPTN/1993/R-B/53)
A. add
B. added
C. addition
D. addictive
E. additional
PEMBAHASAN:
Kalimat tersebut menggunakan modal auxilary can yang harus diikuti oleh verb 1. Jadi,
pilihan A menjadi jawaban yang paling tepat.
Jawaban: A

3. The … child ran into his mother when he heard the thunder. (UMPTN/1992/R-B/34)
A. frightened
B. frighten
C. frightens
D. frightening
E. being frightened

2
PEMBAHASAN:
Kalimat tersebut berarti anak yang … (ketakutan) berlari ke ibunya ketika dia mendengar
suara petir. Kata child merupakan sebuah kata benda (noun), sehingga adjective menjadi
kata yang paling tepat untuk mengikutinya. Jadi, pilihan A menjadi jawaban yang paling
tepat.
Jawaban: A

4. If you want to … in running your business, you must have the full support of your
employees. (UMPTN/1993/R-B/13)
A. succeed
B. successful
C. success
D. successfully
E. succeeded
PEMBAHASAN:
Penggunaan kata to umumnya diikuti oleh infinitive (verb 1). Jadi, pilihan A menjadi
jawaban yang paling tepat.
Jawaban: A

5. The bigger they are, the more powerful the …. (SPMB 2006)
A. attract
B. attraction
C. attracting
D. attractive
E. attracted
PEMBAHASAN:
Kata the adalah determiner. Sebuah determiner umumnya diikuti oleh noun (kata benda).
Jadi, pilihan B menjadi jawaban yang paling tepat.
Jawaban: B

6. The hall has been … decorated for the opening ceremony. (UMPTN/1992/R-B/34)
A. beautiful
B. beauty
C. beautifully
D. beautify
E. beautified

3
PEMBAHASAN:
Has been merupakan salah satu auxiliary verb, sedangkan decorate merupakan kata kerja.
Jenis kata yang dapat diletakkan antara kata kerja bantu dengan kata kerja utama, yaitu
adverb. Jadi, pilihan C menjadi jawaban yang paling tepat.
Jawaban: C

7. The victims of the earthquake showed their … to the local government officials for their
help. (UMPTN/1990/R-B/13)
A. appreciate
B. appreciable
C. appreciation
D. appreciably
E. appreciative
PEMBAHASAN:
Their merupakan kata ganti kepemilikan (possessive pronoun) yang seharusnya diikuti
oleh kata benda (noun). Kata yang paling tepat untuk melengkapinya dengan kata
benda. Jadi, pilihan C menjadi jawaban yang tepat.
Jawaban: C

8. Nowadays, people prefer using small cars than big ones because they are more …
(UMPTN/1991/R-A/63)
A. economically
B. economic
C. economize
D. economical
E. economy
PEMBAHASAN:
Pada kalimat tersebut, penggunaan kata more yang merupakan bagian dari degree of
comparison, biasanya, akan diikuti oleh kata sifat (adjective). Jadi, pilihan D menjadi
jawaban yang paling tepat.
Jawaban: D
9. My uncle … advised me to take English lesson. (UMPTN/1990/R-A/53)
A. strengh
B. strong
C. strengthening
D. strongly
E. strengthened
PEMBAHASAN:
Kalimat tersebut berarti pamansayamenasehatisaya …(dengan sangat) untuk
mengambil les bahasa Inggris. Setelah subjek, ada bagian rumpang yang diikuti oleh
verb (advised).

4
Untuk melengkapi bagian rumpang tersebut, diperlukan adverb. Ini karena umumnya
verb diikuti oleh adverb. Jadi, pilihan D menjadi jawaban yang tepat.
Jawaban: D

10. Anti-nutrients appear to be linked to such things as traffic pollution, antibiotics, deep
fried food, processed meat, and the … of plastic and other chemicals in water. (SPMB
2007)
A. present
B. presently
C. presentation
D. presence
E. presentable
PEMBAHASAN:
The merupakan determiner yang harus diikuti oleh kata benda. Pada pilihan jawaban
tersebut, terdapat dua kata benda, yakni presentation (presentasi) dan presence
(kehadiran). Jadi, pilihan D menjadi jawaban yang paling tepat.
Jawaban: D

5
6

Anda mungkin juga menyukai